Как найти вес жидкости в физике формула

Вы замечали, что предметы в воде становятся легче? Существует много сказок и легенд, когда смекалистые герои поднимали вещи, которые на суше поднять было невозможно.

Мы знаем, что сила тяжести не изменяется, но вес тела может зависеть от множества причин. Когда тело погружается в воду, его сила тяжести не изменяется, но появляется новая сила (открытая Архимедом), которая уменьшает вес этого тела.

  1. Вес тела в воздухе: P=mg. Вес тела направлен вниз.
  2. Архимедова сила: FА=mж⋅g. Сила направлена вверх.
  3. Вес тела в жидкости: P1=P−FА=mg−mжg. Результирующая сил (1) и (2).

Тело, погружённое в жидкость, уменьшается в весе пропорционально весу вытесненной жидкости.

Пример:

определить, сколько весит в воде стеклянная пластина объёмом (1,5) дм³.

Дано Решение

V=1,5дм3=0,0015м3ρж=1000кг/м3ρт=2600кг/м3g=9,8Н/кг

________________

P1=mтg−mжgmт=ρт⋅V;mж=ρж⋅V⇒⇒P1=ρтVg−ρжVg=Vgρт−ρжP1=0,0015м3⋅9,8Н/кг⋅2600кг/м3−1000кг/м3≈24Н

Ответ: стеклянная пластина в воде весит (24) Н.

На прошлом уроке мы доказали с помощью опытов существование силы, действующей на тела, погруженные в жидкость или газ — выталкивающей силы. Также мы теперь знаем, что ее можно рассчитать по формуле: $F_{выт} = gm_ж = P_ж$. Но какое еще есть значение у этой силы? На этом уроке мы более подробно рассмотрим выталкивающую силу.

Выталкивающая сила и вес тела

Как можно на опыте определить, с какой силой тело, погруженное целиком в жидкость, выталкивается из жидкости?
Давайте познакомимся с таким опытом. Он представлен на рисунке 1.

Подвесим на пружину небольшую емкость для жидкости и тело цилиндрической формы ниже. На конце пружины у нас расположена стрелка-указатель. Она отмечает растяжение пружины на штативе (рисунок 1, а). Таким образом, мы видим вес тела в воздухе.

Рисунок 1. Опыт по определению зависимости выталкивающей силы и веса погруженного тела

Теперь опустим наше тело в большой сосуд. Сосуд имеет трубку для слива и наполнен жидкостью до уровня этой трубки (рисунок 1, б).

Когда мы полностью опустим тело в сосуд, часть жидкости из него выльется через трубку для слива в стакан. Объем этой жидкости будет равен объему тела. Мы уже знаем, что на тело действует выталкивающая сила: пружина сокращается, стрелка-указатель поднимается, вес тела в жидкости становится меньше.

А теперь возьмем жидкость, которая вылилась в стакан. Зальем ее в емкость, которая также подвешена к пружине (рисунок 1, в). Теперь стрелка-указатель вернулась к своему изначальному положению.

Так чему равна эта сила? Сделаем вывод из данного опыта.

Сила, выталкивающая целиком погруженное в жидкость тело, равна весу жидкости в объеме этого тела.

Если провести подобный опыт с газом, а не с жидкостью, то мы получим, что сила, выталкивающая тело из газа, равна весу газа, взятого в объеме тела.

Сила Архимеда

Как называют силу, которая выталкивает тела, погруженные в жидкости и газы?
Теперь мы добавим, что эту выталкивающую силу называют архимедовой силой. Архимед (рисунок 2) — древнегреческий ученый и инженер, сделавший множество открытий и в математике, и в физике. Именно он первый обнаружил наличие выталкивающей силы и рассчитал ее значение.

Рисунок 2. Архимед (287–212 годы до н. э.) — древнегреческий ученый и инженер

Как подсчитать архимедову силу?
В прошлом уроке мы получили формулу $F_{выт} = P_ж = g m_ж$. Теперь мы будем называть эту силу архимедовой $F_A$.

Из выше рассмотренных опытов мы можем выразить массу вытесненной жидкости через ее плотность и объем тела, который эту жидкость вытеснил (они одинаковы): $m_ж = rho_ж cdot V_т$. Получим формулу для архимедовой силы.

$F_A = g rho_ж V_т$.

От чего зависит архимедова сила?

Взгляните еще раз на формулу: $F_A = g rho_ж V_т$.

Ясно видно, что архимедова сила зависит только от плотности жидкости и от объема тела, которое мы погружаем в эту жидкость.

Если мы будем погружать в одну и ту же жидкость тела разной плотности и разной формы (рисунок 3), то значение силы меняться не будет (при условии, что эти тела будут обладать одинаковым объемом).

Рисунок 3. Демонстрация равенства силы Архимеда для тел одинакового объема, погруженных в одну и ту же жидкость

Определение веса тела, погруженного в жидкость или газ

На тело, погруженное в жидкость (или в газ), действуют две силы: сила тяжести и архимедова сила. Направлены они в противоположные стороны. Вес тела в жидкости $P_1$ будет меньше веса тела в вакууме $P$ на архимедову силу $F_A$. То есть:
$P_1 = P space − space F_A = gm space − space gm_ж$.

Если тело погружено в жидкость или газ, то его вес уменьшается на вес вытесненной им жидкости или газа.

Пример задачи

Определите выталкивающую силу, которая будет действовать на камень объемом $2.6 space м^3$, лежащий на морском дне.

Дано:
$V_т = 2.6 space м^3$
$rho_ж = 1030 frac{кг}{м^3}$
$g = 9.8 frac{Н}{кг}$

$F_A — ?$

Посмотреть решение и ответ

Скрыть

Решение:

Сила Архимеда рассчитывается по формуле:
$F_A = g rho_ж V_т$.

Подставим численные значения величин и рассчитаем эту силу:
$F_A = 9.8 frac {Н}{кг} cdot 1030 frac{кг}{ м^3} cdot 2.6 space м^3 approx 26 244 space Н approx 26.2 space кН$.

Ответ: $F_A approx 26,2 space кН$.

Забавное дополнение: легенда об Архимеде

Архимед, великий изобретатель, шокировал своих современников гениальными открытиями. Его имя упоминается во множестве легенд, но одна из них стала наиболее известной: легенда о том, как Архимед пришел к открытию выталкивающей силы.

Царь Гиерон поручил Архимеду проверить работу мастера, который изготовил для него золотую корону.

Долгое время ученый не мог найти ответ: как определить количество некачественных примесей? Проблема заключалась в том, что определить ее объем — сложная задача. По легенде озарение настигло Архимеда, когда он принимал ванну.

Ученый заметил, что из ванны вылилась вода, когда он залез в нее. И здесь его посетила гениальная мысль. Все вы слышали его известную цитату: «Эврика! Эврика!» (в переводе означает: «Нашел!  Нашел!»).

Так Архимед победно выкрикивал свою фразу, потрясенный своим открытием, что она дошла в виде легенды и до наших времен.

Упражнения

Упражнение №1

К коромыслу весов подвешены два цилиндра одинаковой массы: свинцовый и алюминиевый (рисунок 4). Весы находятся в равновесии. Нарушится ли равновесие весов, если оба цилиндра одновременно погрузить в воду; в спирт? Ответ обоснуйте. Проверьте его на опыте. Как зависит выталкивающая сила от объема тела?

Рисунок 4. Цилиндры одинаковой массы, но изготовленные из разных материалов

Посмотреть ответ

Скрыть

Ответ:

Когда мы погрузим цилиндры в жидкость, на каждый их них будет действовать сила Архимеда. Если эти силы будут равны, то весы останутся в равновесии.

Запишем формулы архимедовой силы для каждого цилиндра.
Для свинцового цилиндра:
$F_{A1} = g rho_ж V_1$.
Для алюминиевого цилиндра:
$F_{A2} = g rho_ж V_2$.

Мы видим, что равенство этих сил зависит от объемов цилиндров. Они равны? Нет, они имеют одинаковые массы, но разные плотности. Цилиндр из алюминия будет обладать большим объемом, чем свинцовый цилиндр ($V = frac{m}{rho}$). Значит, на алюминиевый цилиндр будет действовать большая выталкивающая сила, чем на свинцовый.

Если мы проверим это на опыте, то увидим подтверждение нашим выводам (рисунок 5).

Рисунок 5. Погружение цилиндров из разных материалов в жидкости

При этом весы выйдут из равновесия в случае и с водой (рисунок 5, а), и со спиртом (рисунок 5, б). Так как мы опускаем цилиндры одновременно в один и тот же тип жидкости, значение архимедовой силы, действующей на цилиндры, будет различаться только в зависимости от объемов этих цилиндров — свинцовый перевесит алюминиевый в любой жидкости.

Заметим, что в случае погружения в воду, архимедова сила будет больше, чем в случае погружения в спирт. Это объясняется тем, что вода имеет большую плотность, чем спирт.

Упражнение №2

К коромыслу весов подвешены два алюминиевых цилиндра одинакового объема. Нарушится ли равновесие весов, если один цилиндр погрузить в воду, а другой — в спирт? Ответ обоснуйте. Зависит ли выталкивающая сила от плотности жидкости?

Посмотреть ответ

Скрыть

Ответ:

Если один цилиндр погрузить в воду, а другой — в спирт, то равновесие весов нарушится (рисунок 6). На цилиндр, находящийся в воде, будет действовать большая архимедова сила.

Рисунок 6. Зависимость величины архимедовой силы от плотности жидкости

Так происходит, потому что архимедова сила зависит от объема погруженного тела (а они у нас одинаковые: $V_1 = V_2 = V$) и от плотности жидкости:
$F_А = g rho_ж V$.
Плотность спирта ($800 frac{кг}{м^3}$) меньше плотности воды ($1000 frac{кг}{м^3}$). Значит, на цилиндр, погруженный в воду, будет действовать большая архимедова сила, чем на тот, что погружен в спирт.

Упражнение №3

Объем куска железа равен $0.1 space дм^3$. Какая выталкивающая сила будет на него действовать при полном его погружении в воду; в керосин?

Дано:
$V = 0.1 space дм^3$
$g = 9.8 frac{Н}{кг}$
$rho_1 = 1000 frac{кг}{м^3}$
$rho_2 = 800 frac{кг}{м^3}$

СИ:
$V = 0.1 cdot 10^{-3} space м^3$

$F_{А1} — ?$
$F_{А2} — ?$

Посмотреть решение и ответ

Скрыть

Решение:

Рассчитаем архимедову силу, которая будет действовать на кусок железа в воде:
$F_{А1} = g rho_1 V$,
$F_{А1} = 9.8 frac{Н}{кг} cdot 1000 frac{кг}{м^3} cdot 0.1 cdot 10^{-3} space м^3 = 0.98 space Н approx 1 space Н$.

Теперь рассчитаем архимедову силу, которая будет действовать на кусок железа в керосине:
$F_{А2} = g rho_2 V$,
$F_{А2} = 9.8 frac{Н}{кг} cdot 800 frac{кг}{м^3} cdot 0.1 cdot 10^{-3} space м^3 = 0.784 space Н approx 0.8 space Н$.

Ответ: $F_{А1} approx 1 space Н$, $F_{А2} approx 0.8 space Н$.

Упражнение №4

Бетонная плита объемом $2 space м^3$ погружена в воду. Какую силу необходимо приложить, чтобы удержать ее в воде; в воздухе?

Дано:
$V = 2 space м^3$
$g = 9.8 frac{Н}{кг}$
$rho_1 = 1000 frac{кг}{м^3}$
$rho_2 = 1.29 frac{кг}{м^3}$
$rho_б = 2300 frac{кг}{м^3}$

$F_1 — ?$
$F_2 — ?$

Посмотреть решение и ответ

Скрыть

Решение:

Бетонная плита находится в воде. На нее действует сила тяжести и архимедова сила. Они направлены противоположно друг другу и будут иметь разные величины. Разность этих сил — и будет искомая сила $F_1$, которую нужно приложить, чтобы удержать бетонную плиту в воде (чтобы она не опускалась на дно и не всплывала):
$F_1 = F_{тяж} space − space F_{А1}$.

Сила тяжести рассчитывается по формуле:
$F_{тяж} = gm$.
Массу бетонной плиты мы можем выразить через ее плотность и объем:
$m = rho_б V$,
$F_{тяж} = g rho_б V$.

Архимедова сила, действующая на бетонную плиту в воде:
$F_{А1} = g rho_1 V$.

Подставим силу тяжести и архимедову силу в формулу и рассчитаем $F_1$:
$F_1 = F_{тяж} space − space F_{А1} = g rho_б V space − space g rho_1 V = gV cdot (rho_б space − space rho_1)$,
$F_1 = 9.8 frac {Н}{кг} cdot 2 space м^3 cdot (2300 frac{кг}{м^3} space − space 1000 frac{кг}{м^3}) = 25 space 480 space Н approx 25 space кН$.

Используем ту же формулу для того, чтобы рассчитать силу $F_2$, которую нужно приложить, чтобы удержать бетонную плиту в воздухе:
$F_2 = gV cdot (rho_б space − space rho_2)$,
$F_2 = 9.8 frac {Н}{кг} cdot 2 space м^3 cdot (2300 frac{кг}{м^3} space − space 1.29 frac{кг}{м^3}) approx 45 space 054 space Н approx 45 space кН$.

Ответ: $F_1 approx 25 space кН$, $F_2 approx 45 space Н$.

Упражнение №5

Предположив, что корона царя Гиерона в воздухе весит $20 space Н$, а в воде — $18.75 space Н$, вычислите плотность вещества короны. Полагая, что к золоту было подмешано только серебро, определите, сколько в короне было золота и сколько серебра. При решении задачи плотность золота считайте равной $20 space 000 frac{кг}{м^3}$, плотность серебра — $10 space 000 frac{кг}{м^3}$. Каков был бы объем короны из чистого золота?

Дано:
$P_1 = 20 space Н$
$P_2 = 18.75 space Н$
$rho_з = 20 space 000 frac{кг}{м^3}$
$rho_с = 10 space 000 frac{кг}{м^3}$
$g = 9.8 frac{Н}{кг}$
$rho_1 = 1.29 frac{кг}{м^3}$
$rho_2 = 1000 frac{кг}{м^3}$

$rho — ?$
$m_з — ?$
$m_с — ?$
$V_1 — ?$

Посмотреть решение и ответ

Скрыть

Решение:

Вес короны в воздухе $P_1$ будет меньше веса тела в вакууме $P$ на архимедову силу $F_{A1}$. То есть:
$P_1 = P space − space F_{A1}$.

Значит, вес короны в вакууме будет равен сумме ее веса в воздухе и архимедовой силы:
$P = P_1 space + space F_{А1}$,
$gm = P_1 space + space g rho_1 V$.

Теперь запишем такое же уравнение для веса короны в воде:
$gm = P_2 space + space g rho_2 V$.

Левые части уравнений у нас равны, поэтому мы можем приравнять правые части друг к другу:
$P_1 space + space g rho_1 V = P_2 space + space g rho_2 V$.
Перенесем элементы, содержащие неизвестный объем вправо:
$P_1 space − space P_2 = g rho_2 V space − space g rho_1 V$,
$P_1 space − space P_2 = gV (rho_2 space − space rho_1)$.

Выразим отсюда объем короны и рассчитаем его:
$V = frac{P_1 space − space P_2}{g (rho_2 space − space rho_1)}$,
$V = frac{20 space Н space − space 18.75 space Н}{9.8 frac{Н}{кг} (1000 frac{кг}{м^3} space − space 1.29 frac{кг}{м^3})} = frac{1.25}{9787} space м^3 = 12.8 cdot 10^{-5} space м^3$.

Используем одно из первых уравнений для веса короны в вакууме и в воздухе:
$gm = P_1 space + space g rho_1 V$.
Выразим отсюда массу короны и рассчитаем ее:
$m = frac{P_1 space + space g rho_1 V}{g}$,
$m = frac{20 space Н space + space 9.8 frac{Н}{кг} cdot 1.29 frac{кг}{м^3} cdot 12.8 cdot 10^{-5} space м^3}{9.8 frac{Н}{кг}} approx 2.04 space кг$.

Теперь мы знаем массу и объем короны. Рассчитаем ее плотность:
$rho = frac{m}{V}$,
$rho = frac{2.04 space кг}{12.8 cdot 10^{-5} space м^3} approx 16 space 000 frac{кг}{м^3}$.

Корона состоит из серебра и золота. Это означает, что ее общий объем мы можем записать в виде суммы объемов серебра и золота, ее составляющих:
$V = V_с space + space V_з$.
То же самое с общей массой короны:
$m = m_с space + space m_з$.

Запишем объемы через массы и плотности (а также выразим массу золота через общую массу короны и массу серебра):
$V_с = frac{m_с}{rho_с}$,
$V_з = frac{m_з}{rho_з} = frac{m space − space m_с}{rho_з}$.

Подставим эти объемы в формулу для общего объема короны и выразим из нее массу серебра:
$V = frac{m_с}{rho_с} space + space frac{m space − space m_с}{rho_з} = frac{m_с (rho_з space − space rho_с) space + space rho_с m}{rho_с rho_з} = m_с cdot frac{rho_з space − space rho_с}{rho_с rho_з} space + space frac{m}{rho_з}$,
$m_с = frac{V space − space frac{m}{rho_з}}{frac{rho_з space − space rho_с}{rho_с rho_з}} = frac{rho_с (V rho_з space − space m)}{rho_з space − space rho_с}$.

Рассчитаем массу серебра, содержащегося в короне:
$m_с = frac{10 space 000 frac{кг}{м^3} (12.8 cdot 10^{-5} space м^3 cdot 20 space 000 frac{кг}{м^3} space − space 2.04 space кг)}{20 space 000 frac{кг}{м^3} space − space 10 space 000 frac{кг}{м^3}} = frac{5200 frac{кг^2}{м^3}}{10 space 000 frac{кг}{м^3}} = 0.52 space кг$.

Теперь мы можем вычислить и количество золота в короне:
$m_з = m space − space m_с$,
$m_з = 2.04 space кг space − space 0.52 space кг = 1.52 space кг$.

Если бы вся корона была из золота, то ее объем был бы равен:
$V_1 = frac{m}{rho_з}$,
$V_1 = frac{2.04 space кг}{20 space 000 frac{кг}{м^3}} = 10.2 cdot 10^{-5} space м^3$.

Ответ: $rho approx 16 space 000 frac{кг}{м^3}$, $m_з = 1.52 space кг$, $m_с = 0.52 space кг$, $V_1 = 10.2 cdot 10^{-5} space м^3$.

Упражнение №6

По мелким камешкам ходить босыми ногами больно. Почему человек не испытывает боли, если ходит по таким же камням в воде?

Посмотреть ответ

Скрыть

Ответ:

Что означает фраза «ходить по камням»? Со стороны физики, когда мы наступаем на камни, мы давим на них своим весом: $p = frac{F}{S} = frac{P}{S}$.

Когда мы оказываемся в воде, наш вес уменьшается. Это следствие действия на нас архимедовой силы. Уменьшается вес — уменьшается и давление наших стоп на камни.

Давление. Единицы давления.

Человек на лыжах, и без них.

По рыхлому снегу человек идёт с большим трудом, глубоко проваливаясь при каждом шаге. Но, надев лыжи, он может идти, почти не проваливаясь в него. Почему? На лыжах или без лыж человек действует на снег с одной и той же силой, равной своему весу. Однако действие этой силы в обоих случаях различно, потому что различна площадь поверхности, на которую давит человек, с лыжами и без лыж. Площадь поверхности лыж почти в 20 раз больше площади подошвы. Поэтому, стоя на лыжах, человек действует на каждый квадратный сантиметр площади поверхности снега с силой, в 20 раз меньшей, чем стоя на снегу без лыж.

Ученик, прикалывая кнопками газету к доске, действует на каждую кнопку с одинаковой силой. Однако кнопка, имеющая более острый конец, легче входит в дерево.

Значит, результат действия силы зависит не только от её модуля, направления и точки приложения, но и от площади той поверхности, к которой она приложена (перпендикулярно которой она действует).

Этот вывод подтверждают физические опыты.

Опыт.Результат действия данной силы зависит от того, какая сила действует на единицу площади поверхности.

По углам небольшой доски надо вбить гвозди. Сначала гвозди, вбитые в доску, установим на песке остриями вверх и положим на доску гирю. В этом случае шляпки гвоздей лишь незначительно вдавливаются в песок. Затем доску перевернем и поставим гвозди на острие. В этом случае площадь опоры меньше, и под действием той же силы гвозди значительно углубляются в песок.

Опыт. Вторая иллюстрация.

От того, какая сила действует на каждую единицу площади поверхности, зависит результат действия этой силы.

В рассмотренных примерах силы действовали перпендикулярно поверхности тела. Вес человека был перпендикулярен поверхности снега; сила, действовавшая на кнопку, перпендикулярна поверхности доски.

Величина, равная отношению силы, действующей перпендикулярно поверхности, к площади этой поверхности, называется давлением.

Чтобы определить давление, надо силу, действующую перпендикулярно поверхности, разделить на площадь поверхности:

давление = сила / площадь.

Обозначим величины, входящие в это выражение: давление — p, сила, действующая на поверхность, — F и площадь поверхности — S.

Тогда получим формулу:

p = F/S

Понятно, что бóльшая по значению сила, действующую на ту же площадь, будет производить большее давление.

За единицу давления принимается такое давление, которое производит сила в 1 Н, действующая на поверхность площадью 1 м2 перпендикулярно этой поверхности.

Единица давления — ньютон на квадратный метр ( 1 Н / м2 ). В честь французского ученого Блеза Паскаля она называется паскалем (Па). Таким образом,

1 Па = 1 Н / м2 .

Используется также другие единицы давления: гектопаскаль (гПа) и килопаскаль (кПа).

1 кПа = 1000 Па;

1 гПа = 100 Па;

1 Па = 0,001 кПа;

1 Па = 0,01 гПа.

Пример. Рассчитать давление, производимое на пол мальчиком, масса которого 45 кг, а площадь подошв его ботинок, соприкасающихся с полом, равна 300 см2.

Запишем условие задачи и решим её.

Дано: m = 45 кг, S = 300 см2; p = ?

В единицах СИ: S = 0,03 м2

Решение:

p = F/S,

F = P,

P = g·m,

P = 9,8 Н · 45 кг ≈ 450 Н,

p = 450/0,03 Н / м2 = 15000 Па = 15 кПа

‘Ответ’: p = 15000 Па = 15 кПа

Способы уменьшения и увеличения давления.

Тяжелый гусеничный трактор производит на почву давление равное 40 — 50 кПа, т. е. всего в 2 — 3 раза больше, чем давление мальчика массой 45 кг. Это объясняется тем, что вес трактора распределяется на бóльшую площадь за счёт гусеничной передачи. А мы установили, что чем больше площадь опоры, тем меньше давление, производимое одной и той же силой на эту опору.

В зависимости от того, нужно ли получить малое или большое давление, площадь опоры увеличивается или уменьшается. Например, для того, чтобы грунт мог выдержать давление возводимого здания, увеличивают площадь нижней части фундамента.

Шины грузовых автомобилей и шасси самолетов делают значительно шире, чем легковых. Особенно широкими делают шины у автомобилей, предназначенных для передвижения в пустынях.

Тяжелые машины, как трактор, танк или болотоход, имея большую опорную площадь гусениц, проходят по болотистой местности, по которой не пройдет человек.

С другой стороны, при малой площади поверхности можно небольшой силой произвести большое давление. Например, вдавливая кнопку в доску, мы действуем на нее с силой около 50 Н. Так как площадь острия кнопки примерно 1 мм2, то давление, производимое ею, равно:

p = 50 Н/ 0, 000 001 м2 = 50 000 000 Па = 50 000 кПа.

Для сравнения, это давление в 1000 раз больше давления, производимого гусеничным трактором на почву. Можно найти еще много таких примеров.

Лезвие режущих и острие колющих инструментов (ножей, ножниц, резцов, пил, игл и др.) специально остро оттачивается. Заточенный край острого лезвия имеет маленькую площадь, поэтому при помощи даже малой силы создается большое давление, и таким инструментом легко работать.

Режущие и колющие приспособления встречаются и в живой природе: это зубы, когти, клювы, шипы и др. — все они из твердого материала, гладкие и очень острые.

Давление

Известно, что молекулы газа беспорядочно движутся.
Опыт. Здесь мы узнаем, что газ давит на стенки сосуда по всем направлениям одинаково.

Мы уже знаем, что газы, в отличие от твердых тел и жидкостей, заполняют весь сосуд, в котором находятся. Например, стальной баллон для хранения газов, камера автомобильной шины или волейбольный мяч. При этом газ оказывает давление на стенки, дно и крышку баллона, камеры или любого другого тела, в котором он находится. Давление газа обусловлено иными причинами, чем давление твердого тела на опору.

Известно, что молекулы газа беспорядочно движутся. При своем движении они сталкиваются друг с другом, а также со стенками сосуда, в котором находится газ. Молекул в газе много, поэтому и число их ударов очень велико. Например, число ударов молекул воздуха, находящегося в комнате, о поверхность площадью 1 см2 за 1 с выражается двадцатитрехзначным числом. Хотя сила удара отдельной молекулы мала, но действие всех молекул на стенки сосуда значительно, — оно и создает давление газа.

Итак, давление газа на стенки сосуда (и на помещенное в газ тело) вызывается ударами молекул газа.

Рассмотрим следующий опыт. Под колокол воздушного насоса поместим резиновый шарик. Он содержит небольшое количество воздуха и имеет неправильную форму. Затем насосом откачиваем воздух из-под колокола. Оболочка шарика, вокруг которой воздух становится все более разреженным, постепенно раздувается и принимает форму правильного шара.

Как объяснить этот опыт?

Для хранения и перевозки сжатого газа используются специальные прочные стальные баллоны.

В нашем опыте движущиеся молекулы газа непрерывно ударяют о стенки шарика внутри и снаружи. При откачивании воздуха число молекул в колоколе вокруг оболочки шарика уменьшается. Но внутри шарика их число не изменяется. Поэтому число ударов молекул о внешние стенки оболочки становится меньше, чем число ударов о внутренние стенки. Шарик раздувается до тех пор, пока сила упругости его резиновой оболочки не станет равной силе давления газа. Оболочка шарика принимает форму шара. Это показывает, что газ давит на ее стенки по всем направлениям одинаково. Иначе говоря, число ударов молекул, приходящихся на каждый квадратный сантиметр площади поверхности, по всем направлениям одинаково. Одинаковое давление по всем направлениям характерно для газа и является следствием беспорядочного движения огромного числа молекул.

Попытаемся уменьшить объем газа, но так, чтобы масса его осталась неизменной. Это значит, что в каждом кубическом сантиметре газа молекул станет больше, плотность газа увеличится. Тогда число ударов молекул о стенки увеличится, т. е. возрастет давление газа. Это можно подтвердить опытом.

На рисунке а изображена стеклянная трубка, один конец которой закрыт тонкой резиновой пленкой. В трубку вставлен поршень. При вдвигании поршня объем воздуха в трубке уменьшается, т. е. газ сжимается. Резиновая пленка при этом выгибается наружу, указывая на то, что давление воздуха в трубке увеличилось.

Наоборот, при увеличении объема этой же массы газа, число молекул в каждом кубическом сантиметре уменьшается. От этого уменьшится число ударов о стенки сосуда — давление газа станет меньше. Действительно, при вытягивании поршня из трубки объем воздуха увеличивается, пленка прогибается внутрь сосуда. Это указывает на уменьшение давления воздуха в трубке. Такие же явления наблюдались бы, если бы вместо воздуха в трубке находился бы любой другой газ.

Итак, при уменьшении объема газа его давление увеличивается, а при увеличении объема давление уменьшается при условии, что масса и температура газа остаются неизменными.

А как изменится давление газа, если нагреть его при постоянном объеме? Известно, что скорость движения молекул газа при нагревании увеличивается. Двигаясь быстрее, молекулы будут ударять о стенки сосуда чаще. Кроме того, каждый удар молекулы о стенку будет сильнее. Вследствие этого, стенки сосуда будут испытывать большее давление.

Следовательно, давление газа в закрытом сосуде тем больше, чем выше температура газа, при условии, что масса газа и объем не изменяются.

Из этих опытов можно сделать общий вывод, что давление газа тем больше, чем чаще и сильнее молекулы ударяют о стенки сосуда.

Для хранения и перевозки газов их сильно сжимают. При этом давление их возрастает, газы необходимо заключать в специальные, очень прочные баллоны. В таких баллонах, например, содержат сжатый воздух в подводных лодках, кислород, используемый при сварке металлов. Конечно же, мы должны навсегда запомнить, что газовые баллоны нельзя нагревать, тем более, когда они заполнены газом. Потому что, как мы уже понимаем, может произойти взрыв с очень неприятными последствиями.

Закон Паскаля.

Давление передается в каждую точку жидкости или газа.
Давление поршня передается в каждую точку жидкости, заполняющей шар.
Теперь газ.

В отличие от твердых тел отдельные слои и мелкие частицы жидкости и газа могут свободно перемещаться относительно друг друга по всем направлениям. Достаточно, например, слегка подуть на поверхность воды в стакане, чтобы вызвать движение воды. На реке или озере при малейшем ветерке появляется рябь.

Подвижностью частиц газа и жидкости объясняется, что давление, производимое на них, передается не только в направлении действия силы, а в каждую точку. Рассмотрим это явление подробнее.

На рисунке, а изображен сосуд, в котором содержится газ (или жидкость). Частицы равномерно распределены по всему сосуду. Сосуд закрыт поршнем, который может перемещаться вверх и вниз.

Прилагая некоторую силу, заставим поршень немного переместиться внутрь и сжать газ (жидкость), находящийся непосредственно под ним. Тогда частицы (молекулы) расположатся в этом месте более плотно, чем прежде(рис, б). Благодаря подвижности частицы газа будут перемещаться по всем направлениям. Вследствие этого их расположение опять станет равномерным, но более плотным, чем раньше (рис, в). Поэтому давление газа всюду возрастет. Значит, добавочное давление передается всем частицам газа или жидкости. Так, если давление на газ (жидкость) около самого поршня увеличится на 1 Па, то во всех точках внутри газа или жидкости давление станет больше прежнего на столько же. На 1 Па увеличится давление и на стенки сосуда, и на дно, и на поршень.

Давление, производимое на жидкость или газ, передается на любую точку одинаково во всех направлениях.

Это утверждение называется законом Паскаля.

На основе закона Паскаля легко объяснить следующие опыты.

На рисунке изображен полый шар, имеющий в различных местах небольшие отверстия. К шару присоединена трубка, в которую вставлен поршень. Если набрать воды в шар и вдвинуть в трубку поршень, то вода польется из всех отверстий шара. В этом опыте поршень давит на поверхность воды в трубке. Частицы воды, находящиеся под поршнем, уплотняясь, передают его давление другим слоям, лежащим глубже. Таким образом, давление поршня передается в каждую точку жидкости, заполняющей шар. В результате часть воды выталкивается из шара в виде одинаковых струек, вытекающих из всех отверстий.

Если шар заполнить дымом, то при вдвигании поршня в трубку из всех отверстий шара начнут выходить одинаковые струйки дыма. Это подтверждает, что и газы передают производимое на них давление во все стороны одинаково.

Давление в жидкости и газе.

Под действием веса жидкости резиновое дно в трубке прогнется.

На жидкости, как и на все тела на Земле, действует сила тяжести. Поэтому, каждый слой жидкости, налитой в сосуд, своим весом создает давление, которое по закону Паскаля передается по всем направлениям. Следовательно, внутри жидкости существует давление. В этом можно убедиться на опыте.

В стеклянную трубку, нижнее отверстие которой закрыто тонкой резиновой пленкой, нальем воду. Под действием веса жидкости дно трубки прогнется.

Опыт показывает, что, чем выше столб воды над резиновой пленкой, тем больше она прогибается. Но всякий раз после того, как резиновое дно прогнулось, вода в трубке приходит в равновесие (останавливается), так как, кроме силы тяжести, на воду действует сила упругости растянутой резиновой пленки.

Иллюстрация.
Дно отходит от цилиндра вследствие давления на него силы тяжести.

Опустим трубку с резиновым дном, в которую налита вода, в другой, более широкий сосуд с водой. Мы увидим, что по мере опускания трубки резиновая пленка постепенно выпрямляется. Полное выпрямление пленки показывает, что силы, действующие на нее сверху и снизу, равны. Наступает полное выпрямление пленки тогда, когда уровни воды в трубке и сосуде совпадают.

Такой же опыт можно провести с трубкой, в которой резиновая пленка закрывает боковое отверстие, как это показано на рисунке, а. Погрузим эту трубку с водой в другой сосуд с водой, как это изображено на рисунке, б. Мы заметим, что пленка снова выпрямится, как только уровни воды в трубке и сосуде сравняются. Это означает, что силы, действующие на резиновую пленку, одинаковы со всех сторон.

Возьмем сосуд, дно которого может отпадать. Опустим его в банку с водой. Дно при этом окажется плотно прижатым к краю сосуда и не отпадет. Его прижимает сила давления воды, направленная снизу вверх.

Будем осторожно наливать воду в сосуд и следить за его дном. Как только уровень воды в сосуде совпадет с уровнем воды в банке, оно отпадет от сосуда.

В момент отрыва на дно давит сверху вниз столб жидкости в сосуде, а снизу вверх на дно передается давление такого же по высоте столба жидкости, но находящейся в банке. Оба эти давления одинаковы, дно же отходит от цилиндра вследствие действия на него собственной силы тяжести.

Выше были описаны опыты с водой, но если взять вместо воды любую другую жидкость, результаты опыта будут те же.

Итак, опыты показывают, что внутри жидкости существует давление, и на одном и том же уровне оно одинаково по всем направлениям. С глубиной давление увеличивается.

Газы в этом отношении не отличаются от жидкостей, ведь они тоже имеют вес. Но надо помнить, что плотность газа в сотни раз меньше плотности жидкости. Вес газа, находящегося в сосуде, мал, и его «весовое» давление во многих случаях можно не учитывать.

Расчет давления жидкости на дно и стенки сосуда.

Расчет давления жидкости на дно и стенки сосуда.

Рассмотрим, как можно рассчитывать давление жидкости на дно и стенки сосуда. Решим сначала задачу для сосуда, имеющего форму прямоугольного параллелепипеда.

Сила F, с которой жидкость, налитая в этот сосуд, давит на его дно, равна весу P жидкости, находящейся в сосуде. Вес жидкости можно определить, зная ее массу m. Массу, как известно, можно вычислить по формуле: m = ρ·V. Объем жидкости, налитой в выбранный нами сосуд, легко рассчитать. Если высоту столба жидкости, находящейся в сосуде, обозначить буквой h, а площадь дна сосуда S, то V = S·h.

Масса жидкости m = ρ·V, или m = ρ·S·h .

Вес этой жидкости P = g·m, или P = g·ρ·S·h.

Так как вес столба жидкости равен силе, с которой жидкость давит на дно сосуда, то, разделив вес P на площадь S, получим давление жидкости p:

p = P/S , или p = g·ρ·S·h/S,

то есть

p = g·ρ·h.

Мы получили формулу для расчета давления жидкости на дно сосуда. Из этой формулы видно, что давление жидкости на дно сосуда зависит только от плотности и высоты столба жидкости.

Следовательно, по выведенной формуле можно рассчитывать давление жидкости, налитой в сосуд любой формы (строго говоря, наш расчет годится только для сосудов, имеющих форму прямой призмы и цилиндра. В курсах физики для института доказано, что формула верна и для сосуда произвольной формы). Кроме того, по ней можно вычислить и давление на стенки сосуда. Давление внутри жидкости, в том числе давление снизу вверх, также рассчитывается по этой формуле, так как давление на одной и той же глубине одинаково по всем направлениям.

При расчете давления по формуле p = gρh надо плотность ρ выражать в килограммах на кубический метр (кг/м3), а высоту столба жидкости h — в метрах (м), g = 9,8 Н/кг, тогда давление будет выражено в паскалях (Па).

Пример. Определите давление нефти на дно цистерны, если высота столба нефти 10 м, а плотность ее 800 кг/м3 .

Запишем условие задачи и запишем ее.

Дано:

h = 10 м

ρ = 800 кг/м3

P = ?

Решение:

p = gρh,

p = 9.8 Н/кг · 800 кг/м3 · 10 м ≈ 80 000 Па ≈ 80 кПа.

Ответ: p ≈ 80 кПа.

Сообщающиеся сосуды.

Сообщающиеся сосуды.

На рисунке изображены два сосуда, соединённые между собой резиновой трубкой. Такие сосуды называются сообщающимися. Лейка, чайник, кофейник — примеры сообщающихся сосудов. Из опыта мы знаем, что вода, налитая, например, в лейку, стоит всегда на одном уровне в носике и внутри.

С сообщающимися сосудами можно проделать следующий простой опыт. В начале опыта резиновую трубку зажимаем в середине, и в одну из трубок наливаем воду. Затем зажим открываем, и вода вмиг перетекает в другую трубку, пока поверхности воды в обеих трубках не установятся на одном уровне. Можно закрепить одну из трубок в штативе, а другую поднимать, опускать или наклонять в разные стороны. И в этом случае, как только жидкость успокоится, ее уровни в обеих трубках уравняются.

В сообщающихся сосудах любой формы и сечения поверхности однородной жидкости устанавливаются на одном уровне (при условии, что давление воздуха над жидкостью одинаково) (рис. 109).

Это можно обосновать следующим образом. Жидкость покоится, не перемещаясь из одного сосуда в другой. Значит, давления в обоих сосудах на любом уровне одинаковы. Жидкость в обоих сосудах одна и та же, т. е. имеет одинаковую плотность. Следовательно, должны быть одинаковы и ее высоты. Когда мы поднимаем один сосуд или доливаем в него жидкость, давление в нем увеличивается и жидкость перемещается в другой сосуд до тех пор, пока давления не уравновесятся.

Если в один из сообщающихся сосудов налить жидкость одной плотности, а во второй — другой плотности, то при равновесии уровни этих жидкостей не будут одинаковыми. И это понятно. Мы ведь знаем, что давление жидкости на дно сосуда прямо пропорционально высоте столба и плотности жидкости. А в этом случае плотности жидкостей будут различны.

При равенстве давлений высота столба жидкости с большей плотностью будет меньше высоты столба жидкости с меньшей плотностью (рис.).

Опыт. Как определить массу воздуха.

Вес воздуха. Атмосферное давление.

Существование атмосферного давления.
Атмосферное давление больше, чем давление разреженного воздуха в сосуде.

На воздух, как и на всякое тело, находящееся на Земле, действует сила тяжести, и, значит, воздух обладает весом. Вес воздуха легко вычислить, зная его массу.

На опыте покажем, как вычислить массу воздуха. Для этого нужно взять прочный стеклянный шар с пробкой и резиновой трубкой с зажимом. Выкачаем из него насосом воздух, зажмем трубку зажимом и уравновесим на весах. Затем, открыв зажим на резиновой трубке, впустим в него воздух. Равновесие весов при этом нарушится. Для его восстановления на другую чашку весов придется положить гири, масса которых будет равна массе воздуха в объеме шара.

Опытами установлено, что при температуре 0 °С и нормальном атмосферном давлении масса воздуха объемом 1 м3 равна 1,29 кг. Вес этого воздуха легко вычислить:

P = g·m, P = 9,8 Н/кг · 1,29 кг ≈ 13 Н.

Воздушная оболочка, окружающая Землю, называется атмосфера (от греч. атмос — пар, воздух, и сфера — шар).

Атмосфера, как показали наблюдения за полетом искусственных спутников Земли, простирается на высоту нескольких тысяч километров.

Вследствие действия силы тяжести верхние слои атмосферы, подобно воде океана, сжимают нижние слои. Воздушный слой, прилегающий непосредственно к Земле, сжат больше всего и, согласно закону Паскаля, передает производимое на него давление по всем направлениям.

В результате этого земная поверхность и телá, находящиеся на ней, испытывают давление всей толщи воздуха, или, как обычно говорится в таких случаях, испытывают атмосферное давление.

Существованием атмосферного давления могут быть объяснены многие явления, с которыми мы встречаемся в жизни. Рассмотрим некоторые из них.

На рисунке изображена стеклянная трубка, внутри которой находится поршень, плотно прилегающий к стенкам трубки. Конец трубки опущен воду. Если поднимать поршень, то за ним будет подниматься и вода.

Это явление используется в водяных насосах и некоторых других устройствах.

На рисунке показан цилиндрический сосуд. Он закрыт пробкой, в которую вставлена трубка с краном. Из сосуда насосом откачивается воздух. Затем конец трубки помещается в воду. Если теперь открыть кран, то вода фонтаном брызнет в внутрь сосуда. Вода поступает в сосуд потому, что атмосферное давление больше давления разреженного воздуха в сосуде.

Почему существует воздушная оболочка Земли.

Как и все тела, молекулы газов, входящих в состав воздушной оболочки Земли, притягиваются к Земле.

Но почему же тогда все они не упадут на поверхность Земли? Каким образом сохраняется воздушная оболочка Земли, ее атмосфера? Чтобы понять это, надо учесть, что молекулы газов находятся в непрерывном и беспорядочном движении. Но тогда возникает другой вопрос: почему эти молекулы не улетают в мировое пространство, то есть в космос.

Для того, чтобы совсем покинуть Землю, молекула, как и космический корабль или ракета, должна иметь очень большую скорость (не меньше 11,2 км/с). Это так называемая вторая космическая скорость. Скорость большинства молекул воздушной оболочки Земли значительно меньше этой космической скорости. Поэтому большинство их привязано к Земле силой тяжести, лишь ничтожно малое количество молекул улетает за пределы Земли в космос.

Беспорядочное движение молекул и действие на них силы тяжести приводят в результате к тому, что молекулы газов «парят» в пространстве около Земли, образуя воздушную оболочку, или известную нам атмосферу.

Измерения показывают, что плотность воздуха быстро уменьшается с высотой. Так, на высоте 5,5 км над Землей плотность воздуха в 2 раза меньше его плотность у поверхности Земли, на высоте 11 км — в 4 раза меньше, и т. д. Чем выше, тем воздух разреженнее. И наконец, в самых верхних слоях (сотни и тысячи километров над Землей) атмосфера постепенно переходит в безвоздушное пространство. Четкой границы воздушная оболочка Земли не имеет.

Строго говоря, вследствие действия силы тяжести плотность газа в любом закрытом сосуде неодинакова по всему объему сосуда. Внизу сосуда плотность газа больше, чем в верхних его частях, поэтому и давление в сосуде неодинаково. На дне сосуда оно больше, чем вверху.
Однако для газа, содержащегося в сосуде, это различие в плотности и давлении столь мало, что его можно во многих случаях совсем не учитывать, просто знать об этом. Но для атмосферы, простирающейся на несколько тысяч километров, различие это существенно.

Измерение атмосферного давления. Опыт Торричелли.

Рассчитать атмосферное давление по формуле для вычисления давления столба жидкости (§ 38) нельзя. Для такого расчета надо знать высоту атмосферы и плотность воздуха. Но определенной границы у атмосферы нет, а плотность воздуха на разной высоте различна. Однако измерить атмосферное давление можно с помощью опыта, предложенного в 17 веке итальянским ученым Эванджелиста Торричелли, учеником Галилея.

Опыт Торричелли состоит в следующем: стеклянную трубку длиной около 1 м, запаянную с одного конца, наполняют ртутью. Затем, плотно закрыв второй конец трубки, ее переворачивают и опускают в чашку с ртутью, где под уровнем ртути открывают этот конец трубки. Как и в любом опыте с жидкостью, часть ртути при этом выливается в чашку, а часть ее остается в трубке. Высота столба ртути, оставшейся в трубке, равна примерно 760 мм. Над ртутью внутри трубки воздуха нет, там безвоздушное пространство, поэтому никакой газ не оказывает давления сверху на столб ртути внутри этой трубки и не влияет на измерения.

Опыт Торричелли.

Торричелли, предложивший описанный выше опыт, дал и его объяснение. Атмосфера давит на поверхность ртути в чашке. Ртуть находится в равновесии. Значит, давление в трубке на уровне аа1 (см. рис) равно атмосферному давлению. При изменении атмосферного давления меняется и высота столба ртути в трубке. При увеличении давления столбик удлиняется. При уменьшении давления — столб ртути уменьшает свою высоту.

Давление в трубке на уровне аа1 создается весом столба ртути в трубке, так как в верхней части трубки над ртутью воздуха нет. Отсюда следует, что атмосферное давление равно давлению столба ртути в трубке, т. е.

pатм = pртути .

Измерив высоту столба ртути, можно рассчитать давление, которое производит ртуть. Оно и будет равно атмосферному давлению. Если атмосферное давление уменьшится, то столб ртути в трубке Торричелли понизится.

Чем больше атмосферное давление, тем выше столб ртути в опыте Торричелли. Поэтому на практике атмосферное давление можно измерить высотой ртутного столба (в миллиметрах или сантиметрах). Если, например, атмосферное давление равно 780 мм рт. ст. (говорят «миллиметров ртутного столба»), то это значит, что воздух производит такое же давление, какое производит вертикальный столб ртути высотой 780 мм.

Следовательно, в этом случае за единицу измерения атмосферного давления принимается 1 миллиметр ртутного столба (1 мм рт. ст.). Найдем соотношение между этой единицей и известной нам единицей — паскалем (Па).

Давление столба ртути ρртути высотой 1 мм равно:

p = g·ρ·h, p = 9,8 Н/кг · 13 600 кг/ м3 · 0,001 м ≈ 133,3 Па.

Итак, 1 мм рт. ст. = 133,3 Па.

В настоящее время атмосферное давление принято измерять в гектопаскалях ( 1 гПа = 100 Па). Например, в сводках погоды может быть объявлено, что давление равно 1013 гПа, это то же самое, что 760 мм рт. ст.

Наблюдая ежедневно за высотой ртутного столба в трубке, Торричелли обнаружил, что эта высота меняется, т. е. атмосферное давление непостоянно, оно может увеличиваться и уменьшаться. Торричелли заметил также, что атмосферное давление связано с изменением погоды.

Если к трубке с ртутью, использовавшейся в опыте Торричелли, прикрепить вертикальную шкалу, то получится простейший прибор — ртутный барометр (от греч. барос — тяжесть, метрео — измеряю). Он служит для измерения атмосферного давления.

Барометр — анероид.

В практике для измерения атмосферного давления используют металлический барометр, называемый анероидом (в переводе с греческого — безжидкостный). Так барометр называют потому, что в нем нет ртути.

Внешний вид анероида изображен на рисунке. Главная часть его — металлическая коробочка 1 с волнистой (гофрированной) поверхностью (см. др. рис.). Из этой коробочки выкачан воздух, а чтобы атмосферное давление не раздавило коробочку, ее крышка 2 пружиной оттягивается вверх. При увеличении атмосферного давления крышка прогибается вниз и натягивает пружину. При уменьшении давления пружина выпрямляет крышку. К пружине с помощью передаточного механизма 3 прикреплена стрелка-указатель 4, которая продвигается вправо или влево при изменении давления. Под стрелкой укреплена шкала, деления которой нанесены по показаниям ртутного барометра. Так, число 750, против которого стоит стрелка анероида (см. рис.), показывает, что в данный момент в ртутном барометре высота ртутного столба 750 мм.

Следовательно, атмосферное давление равно 750 мм рт. ст. или ≈ 1000 гПа.

Значение атмосферного давления весьма важно для предвидения погоды на ближайшие дни, так как изменение атмосферного давления связано с изменением погоды. Барометр — необходимый прибор для метеорологических наблюдений.

Атмосферное давление на различных высотах.

В жидкости давление, как мы знаем, зависит от плотности жидкости и высоты ее столба. Вследствие малой сжимаемости плотность жидкости на различных глубинах почти одинакова. Поэтому, вычисляя давление, мы считаем ее плотность постоянной и учитываем только изменение высоты.

Сложнее дело обстоит с газами. Газы сильно сжимаемы. А чем сильнее газ сжат, тем больше его плотность, и тем большее давление он производит. Ведь давление газа создается ударами его молекул о поверхность тела.

Слои воздуха у поверхности Земли сжаты всеми вышележащими слоями воздуха, находящимися над ними. Но чем выше от поверхности слой воздуха, тем слабее он сжат, тем меньше его плотность. Следовательно, тем меньшее давление он производит. Если, например, воздушный шар поднимается над поверхностью Земли, то давление воздуха на шар становиться меньше. Это происходит не только потому, что высота столба воздуха над ним уменьшается, но еще и потому, что уменьшается плотность воздуха. Вверху она меньше, чем внизу. Поэтому зависимость давления воздуха от высоты сложнее, чем жидкости.

Наблюдения показывают, что атмосферное давление в местностях, лежащих на уровне моря, в среднем равно 760 мм рт. ст.

Атмосферное давление, равное давлению столба ртути высотой 760 мм при температуре 0 °С, называется нормальным атмосферным давлением.

Нормальное атмосферное давление равно 101 300 Па = 1013 гПа.

Чем больше высота над уровнем моря, тем давление меньше.

При небольших подъемах, в среднем, на каждые 12 м подъема давление уменьшается на 1 мм рт. ст. (или на 1,33 гПа).

Зная зависимость давления от высоты, можно по изменению показаний барометра определить высоту над уровнем моря. Анероиды, имеющие шкалу, по которой непосредственно можно измерить высоту над уровнем моря, называются высотомерами. Их применяют в авиации и при подъеме на горы.

Манометры.

Мы уже знаем, что для измерения атмосферного давления применяют барометры. Для измерения давлений, бóльших или меньших атмосферного, используется манометры (от греч. манос — редкий, неплотный, метрео — измеряю). Манометры бывают жидкостные и металлические.

Рассмотрим сначала устройство и действие открытого жидкостного манометра. Он состоит из двухколенной стеклянной трубки, в которую наливается какая-нибудь жидкость. Жидкость устанавливается в обоих коленах на одном уровне, так как на ее поверхность в коленах сосуда действует только атмосферное давление.

Чтобы понять, как работает такой манометр, его можно соединить резиновой трубкой с круглой плоской коробкой, одна сторона которой затянута резиновой пленкой. Если надавить пальцем на пленку, то уровень жидкости в колене манометра, соединенном в коробкой, понизится, а в другом колене повысится. Чем это объясняется?

При надавливании на пленку увеличивается давление воздуха в коробке. По закону Паскаля это увеличение давления передается и жидкости в том колене манометра, которое присоединено к коробке. Поэтому давление на жидкость в этом колене будет больше, чем в другом, где на жидкость действует только атмосферное давление. Под действием силы этого избыточного давления жидкость начнет перемещаться. В колене со сжатым воздухом жидкость опустится, в другом — поднимется. Жидкость придет в равновесие (остановится), когда избыточное давление сжатого воздуха уравновесится давлением, которое производит избыточный столб жидкости в другом колене манометра.

Чем сильнее давить на пленку, тем выше избыточный столб жидкости, тем больше его давление. Следовательно, об изменении давления можно судить по высоте этого избыточного столба.

На рисунке показано, как таким манометром можно измерять давление внутри жидкости. Чем глубже погружается в жидкость трубочка, тем больше становится разность высот столбов жидкости в коленах манометра, тем, следовательно, и большее давление производит жидкость.

Если установить коробочку прибора на какой-нибудь глубине внутри жидкости и поворачивать ее пленкой вверх, вбок и вниз, то показания манометра при этом не будут меняется. Так и должно быть, ведь на одном и том же уровне внутри жидкости давление одинаково по всем направлениям.

На рисунке изображен металлический манометр. Основная часть такого манометра — согнутая в трубу металлическая трубка 1, один конец которой закрыт. Другой конец трубки с помощью крана 4 сообщается с сосудом, в котором измеряют давление. При увеличении давления трубка разгибается. Движение её закрытого конца при помощи рычага 5 и зубчатки 3 передается стрелке 2, движущейся около шкалы прибора. При уменьшении давления трубка, благодаря своей упругости, возвращается в прежнее положение, а стрелка — к нулевому делению шкалы.

Поршневой жидкостный насос.

В опыте, рассмотренном нами ранее (§ 40), было установлено, что вода в стеклянной трубке под действием атмосферного давления поднималась вверх за поршнем. На этом основано действие поршневых насосов.

Насос схематически изображен на рисунке. Он состоит из цилиндра, внутри которого ходит вверх и вниз, плотно прилегая к стенкам сосуда, поршень 1. В нижней части цилиндра и в самом поршне установлены клапаны 2, открывающиеся только вверх. При движении поршня вверх вода под действием атмосферного давления входит в трубу, поднимает нижний клапан и движется за поршнем.

При движении поршня вниз вода, находящаяся под поршнем, давит на нижний клапан, и он закрывается. Одновременно под давлением воды открывается клапан внутри поршня, и вода переходит в пространство над поршнем. При следующем движении поршня вверх в месте с ним поднимается и находящаяся над ним вода, которая и выливается в отводящую трубу. Одновременно за поршнем поднимается и новая порция воды, которая при последующем опускании поршня окажется над ним, и вся эта процедура повторяется вновь и вновь, пока работает насос.

Гидравлический пресс.

Закон Паскаля позволяет объяснить действие гидравлической машины (от греч. гидравликос — водяной). Это машины, действие которых основано на законах движения и равновесия жидкостей.

Основной частью гидравлической машины служат два цилиндра разного диаметра, снабженные поршнями и соединительной трубкой. Пространство под поршнями и трубку заполняют жидкостью (обычно минеральным маслом). Высоты столбов жидкости в обоих цилиндрах одинаковы, пока на поршни не действуют силы.

Допустим теперь, что силы F1 и F2 — силы, действующие на поршни, S1 и S2 — площади поршней. Давление под первым (малым) поршнем равно p1 = F1 / S1, а под вторым (большим) p2 = F2 / S2 . По закону Паскаля давление покоящейся жидкостью во все стороны передается одинаково, т. е. p1 = p2 или F1 / S1 = F2 / S2 , откуда:

F2 / F1 = S2 / S1 .

Следовательно, сила F2 во столько раз больше силы F1 , во сколько раз площадь большого поршня больше площади малого поршня. Например, если площадь большого поршня 500 см2, а малого 5 см2, и на малый поршень действует сила 100 Н, то на больший поршень будет действовать сила, в 100 раз бóльшая, то есть 10 000 Н.

Таким образом, с помощью гидравлической машины можно малой силой уравновесить бóльшую силу.

Отношение F1 / F2 показывает выигрыш в силе. Например, в приведенном примере выигрыш в силе равен 10 000 Н / 100 Н = 100.

Гидравлическая машина, служащая для прессования (сдавливания), называется гидравлическим прессом.

Гидравлические прессы применяются там, где требуется большая сила. Например, для выжимания масла из семян на маслобойных заводах, для прессования фанеры, картона, сена. На металлургических заводах гидравлические прессы используют для изготовления стальных валов машин, железнодорожных колес и многих других изделий. Современные гидравлические прессы могут развивать силу в десятки и сотни миллионов ньютонов.

Устройство гидравлического пресса схематически показано на рисунке. Прессуемое тело 1 (A) кладут на платформу, соединенную с большим поршнем 2 (B). При помощи малого поршня 3 (D) создается большое давление на жидкость. Это давление передается в каждую точку жидкости, заполняющей цилиндры. Поэтому такое же давление действует и на второй, большой поршень. Но так как площадь 2-го (большого) поршня больше площади малого, то и сила, действующая на него, будет больше силы, действующей на поршень 3 (D). Под действием этой силы поршень 2 (B) будет подниматься. При подъеме поршня 2 (B) тело (A) упирается в неподвижную верхнюю платформу и сжимается. При помощи манометра 4 (M) измеряется давление жидкости. Предохранительный клапан 5 (P) автоматически открывается, когда давление жидкости превышает допустимое значение.

Из малого цилиндра в большой жидкость перекачивается повторными движениями малого поршня 3 (D). Это осуществляется следующим образом. При подъеме малого поршня (D) клапан 6 (K) открывается, и в пространство, находящееся под поршнем, засасывается жидкость. При опускании малого поршня под действием давления жидкости клапан 6 (K) закрывается, а клапан 7 (K’) открывается, и жидкость переходит в большой сосуд.

Действие воды и газа на погруженное в них тело.

Под водой мы легко можем поднять камень, который с трудом поднимается в воздухе. Если погрузить пробку под воду и выпустить ее из рук, то она всплывет. Как можно объяснить эти явления?

Мы знаем (§ 38), что жидкость давит на дно и стенки сосуда. И если внутрь жидкости поместить какое-нибудь твердое тело, то оно также будет подвергаться давлению, как и стенки сосуда.

Рассмотрим силы, которые действуют со стороны жидкости на погруженное в нее тело. Чтобы легче было рассуждать, выберем тело, которое имеет форму параллелепипеда с основаниями, параллельными поверхности жидкости (рис.). Силы, действующие на боковые грани тела, попарно равны и уравновешивают друг друга. Под действием этих сил тело сжимается. А вот силы, действующие на верхнюю и нижнюю грани тела, неодинаковы. На верхнюю грань давит сверху силой F1 столб жидкости высотой h1 . На уровне нижней грани давление производит столб жидкости высотой h2. Это давление, как мы знаем (§ 37), передается внутри жидкости во все стороны. Следовательно, на нижнюю грань тела снизу вверх с силой F2 давит столб жидкости высотой h2. Но h2 больше h1, следовательно, и модуль силы F2 больше модуля силы F1. Поэтому тело выталкивается из жидкости с силой Fвыт, равной разности сил F2F1 , т. е.

Fвыт = F2F1

Рассчитаем эту выталкивающую силу. Силы F1 и F2 , действующие на верхнюю и нижнюю грани параллелепипеда, можно вычислить, зная площади этих граней (S1 и S2) и давление жидкости на уровнях этих граней (p1 и p2):

F1 = p1·S1, а F2 = p2·S2, так как p1 = ρж·g·h1 , p2 = ρж·g·h2 , а S1 = S2 = S, где S — площадь грани параллелепипеда (все грани равны).

Тогда, Fвыт = F2 — F1 = ρ·g·h2·S — ρ·g·h1·S = ρ·g·S·(h2 — h1) = ρ·g·S·h, где h — высота параллелепипеда (h = h2 — h1).

Но S·h = V, где V — объем параллелепипеда, а ρж·V = mж — масса жидкости в объеме параллелепипеда. Следовательно,

Fвыт = g·mж = Pж ,

т. е. выталкивающая сила равна весу жидкости в объеме погруженного в нее тела (выталкивающая сила равна весу жидкости такого же объёма, как и объём погруженного в нее тела).

Существование силы, выталкивающей тело из жидкости, легко обнаружить на опыте.

На рисунке а изображено тело, подвешенное к пружине со стрелкой-указателем на конце. Стрелка отмечает на штативе растяжение пружины. При отпускании тела в воду пружина сокращается (рис., б). Такое же сокращение пружины получится, если действовать на тело снизу вверх с некоторой силой, например, нажать рукой (приподнять).

Следовательно, опыт подтверждает, что на тело, находящееся в жидкости, действует сила, выталкивающая это тело из жидкости.

К газам, как мы знаем, также применим закон Паскаля. Поэтому на тела, находящиеся в газе, действует сила, выталкивающая их из газа. Под действием этой силы воздушные шары поднимаются вверх. Существование силы, выталкивающей тело из газа, можно также наблюдать на опыте.

К укороченной чашке весов подвесим стеклянный шар или большую колбу, закрытую пробкой. Весы уравновешиваются. Затем под колбу (или шар) ставят широкий сосуд так, чтобы он окружал всю колбу. Сосуд наполняется углекислым газом, плотность которого больше плотности воздуха (поэтому углекислый газ опускается вниз и заполняет сосуд, вытесняя из него воздух). При этом равновесие весов нарушается. Чашка с подвешенной колбой поднимается вверх (рис.). На колбу, погруженную в углекислый газ, действует бóльшая выталкивающая сила, по сравнению с той, которая действует на нее в воздухе.

Сила, выталкивающая тело из жидкости или газа, направлена противоположно силе тяжести, приложенной к этому телу.

Поэтому пролкосмосе). Именно этим объясняется, что в воде мы иногда легко поднимаем тела, которые с трудом удерживаем в воздухе.

Архимедова сила.

Силу, с которой тело, находящееся в жидкости, выталкивается ею, можно рассчитать (как это сделано в § 48). А можно определить ее значение на опыте, используя для этого прибор, изображенный на рисунке.

К пружине подвешивается небольшое ведерко и тело цилиндрической формы (рис., а). Стрелка на штативе отмечает растяжение пружины. Она показывает вес тела в воздухе. Приподняв тело, под него подставляется отливной сосуд, наполненный жидкостью до уровня отливной трубки. После чего тело погружается целиком в жидкость (рис., б). При этом часть жидкости, объем которой равен объему тела, выливается из отливного сосуда в стакан. Пружина сокращается, и указатель пружины поднимается вверх, показывая уменьшение веса тела в жидкости. В данном случае на тело, кроме силы тяжести, действует еще одна сила, выталкивающая его из жидкости. Если в верхнее ведерко вылить жидкость из стакана (т. е. ту, которую вытеснило тело), то указатель пружины возвратится к своему начальному положению (рис., в).

На основании этого опыта можно заключить, что сила, выталкивающая целиком погруженное в жидкость тело, равна весу жидкости в объеме этого тела. Такой же вывод мы получили и в § 48.

Если подобный опыт проделать с телом, погруженным в какой-либо газ, то он показал бы, что сила, выталкивающая тело из газа, также равна весу газа, взятого в объеме тела.

Сила, выталкивающая тело из жидкости или газа, называется архимедовой силой, в честь ученого Архимеда, который впервые указал на ее существование и рассчитал ее значение.

Итак, опыт подтвердил, что архимедова (или выталкивающая) сила равна весу жидкости в объеме тела, т. е. FА = Pж = g·mж. Массу жидкости mж, вытесняемую телом, можно выразить через ее плотность ρж и объем тела Vт, погруженного в жидкость (так как Vж — объем вытесненной телом жидкости равен Vт — объему тела, погруженного в жидкость), т. е. mж = ρж·Vт. Тогда получим:

FA = g·ρж·Vт

Следовательно, архимедова сила зависит от плотности жидкости, в которую погружено тело, и от объема этого тела. Но она не зависит, например, от плотности вещества тела, погружаемого в жидкость, так как эта величина не входит в полученную формулу.

Определим теперь вес тела, погруженного в жидкость (или в газ). Так как две силы, действующие на тело в этом случае, направлены в противоположные стороны (сила тяжести вниз, а архимедова сила вверх), то вес тела в жидкости P1 будет меньше веса тела в вакууме P = g·m на архимедову силу FА = g·mж (где mж — масса жидкости или газа, вытесненной телом).

Таким образом, если тело погружено в жидкость или газ, то оно теряет в своем весе столько, сколько весит вытесненная им жидкость или газ.

Пример. Определить выталкивающую силу, действующую на камень объемом 1,6 м3 в морской воде.

Запишем условие задачи и решим ее.

Дано:

Vт =1,6 м3

ρж = 1030 кг/м3

g = 9,8 Н/кг

FА — ?

Решение:

FА = g·ρж·Vт,

FА = 9.8 Н/кг · 1030 кг/м3 · 1,6 м3 = 16 480 Н ≈ 16,5 кН.

Ответ: FА = 16,5 кН.

Плавание тел.

На тело, находящееся внутри жидкости, действуют две силы: сила тяжести, направленная вертикально вниз, и архимедова сила, направленная вертикально вверх. Рассмотрим, что будет происходить с телом под действием этих сил, если в начале оно было неподвижно. При этом возможны три случая:

1) если сила тяжести Fтяж больше архимедовой силы FА, то тело будет опускаться на дно, тонуть, т. е. если

Fтяж > FА, то тело тонет;

2) если сила тяжести равна архимедовой силе, то тело может находиться в равновесии в любом месте жидкости, т. е. если

Fтяж = FА, то тело плавает;

3) если сила тяжести меньше архимедовой силы, то тело будет подниматься из жидкости, всплывать, т. е. если

Fтяж < FА, то тело всплывает.

Рассмотрим последний случай подробнее.

Когда всплывающее тело достигнет поверхности жидкости, то при дальнейшем его движении вверх архимедова сила будет уменьшаться. Почему? А потому, что будет уменьшаться объем части тела, погруженной в жидкость, а архимедова сила равна весу жидкости в объеме погруженной в нее части тела.

Когда архимедова сила станет равной силе тяжести, тело остановится и будет плавать на поверхности жидкости, частично погрузившись в нее.

Полученный вывод легко проверить на опыте.

В отливной сосуд нальем воду до уровня отливной трубки. После этого погрузим в сосуд плавающее тело, предварительно взвесив его в воздухе. Опустившись в воду, тело вытесняет объем воды, равный объему погруженной в нее части тела. Взвесив эту воду, находим, что ее вес (архимедова сила) равен силе тяжести, действующей на плавающее тело, или весу этого тела в воздухе.

Проделав такие же опыты с любыми другими телами, плавающими в разных жидкостях — в воде, спирте, растворе соли, можно убедиться, что если тело плавает в жидкости, то вес вытесненной им жидкости равен весу этого тела в воздухе.

Легко доказать, что если плотность сплошного твердого тела больше плотности жидкости, то тело в такой жидкости тонет. Тело с меньшей плотностью всплывает в этой жидкости. Кусок железа, например, тонет в воде, но всплывает в ртути. Тело же, плотность которого равна плотности жидкости, остается в равновесии внутри жидкости.

Плавает на поверхности воды лед, так как его плотность меньше плотности воды.

Чем меньше плотность тела по сравнению с плотностью жидкости, тем меньшая часть тела погружена в жидкость.

При равных плотностях тела и жидкости тело плавает внутри жидкости на любой глубине.

Две несмешивающиеся жидкости, например вода и керосин, располагаются в сосуде в соответствии со своими плотностями: в нижней части сосуда — более плотная вода (ρ = 1000 кг/м3), сверху — более легкий керосин (ρ = 800 кг/м3).

Средняя плотность живых организмов, населяющих водную среду, мало отличается от плотности воды, поэтому их вес почти полностью уравновешивается архимедовой силой. Благодаря этому водные животные не нуждаются в столь прочных и массивных скелетах, как наземные. По этой же причине эластичны стволы водных растений.

Плавательный пузырь рыбы легко меняет свой объем. Когда рыба с помощью мышц опускается на большую глубину, и давление воды на нее увеличивается, пузырь сжимается, объем тела рыбы уменьшается, и она не выталкивается вверх, а плавает в глубине. Таким образом, рыба может в определенных пределах регулировать глубину своего погружения. Киты регулируют глубину своего погружения за счет уменьшения и увеличения объема легких.

Плавание судов.

Суда, плавающие по рекам, озерам, морям и океанам, построены из разных материалов с различной плотностью. Корпус судов обычно делается из стальных листов. Все внутренние крепления, придающие судам прочность, также изготовляют из металлов. Для постройки судов используют различные материалы, имеющие по сравнению с водой как бóльшие, так и меньшие плотности.

Благодаря чему суда держатся на воде, принимают на борт и перевозят большие грузы?

Опыт с плавающим телом (§ 50) показал, что тело вытесняет своей подводной частью столько воды, что по весу эта вода равна весу тела в воздухе. Это также справедливо и для любого судна.

Вес воды, вытесняемой подводной частью судна, равен весу судна с грузом в воздухе или силе тяжести, действующей на судно с грузом.

Глубина, на которую судно погружается в воду, называется осадкой. Наибольшая допускаемая осадка отмечена на корпусе судна красной линией, называемой ватерлинией (от голланд. ватер — вода).

Вес воды, вытесняемой судном при погружении до ватерлинии, равный силе тяжести, действующей на судно с грузом, называется водоизмещением судна.

В настоящее время для перевозки нефти строятся суда водоизмещением 5 000 000 кН (5 · 106 кН) и больше, т. е. имеющие вместе с грузом массу 500 000 т (5 · 105 т) и более.

Если из водоизмещения вычесть вес самого судна, то мы получим грузоподъемность этого судна. Грузоподъемность показывает вес груза, перевозимого судном.

Судостроение существовало еще в Древнем Египте, в Финикии (считается, что Финикийцы были одними из лучших судостроителей), Древнем Китае.

В России судостроение зародилось на рубеже 17-18 вв. Сооружались главным образом военные корабли, но именно в России были построены первый ледокол, суда с двигателем внутреннего сгорания, атомный ледокол «Арктика».

Воздухоплавание.

Рисунок с описанием шара братьев Монгольфье 1783 года: «Вид и точные размеры „Аэростата Земной шар“, который был первым». 1786

С давних времен люди мечтали о возможности летать над облаками, плавать в воздушном океане, как они плавали по морю. Для воздухоплавания

вначале использовали воздушные шары, которые наполняли или нагретым воздухом, или водородом либо гелием.

Для того, чтобы воздушный шар поднялся в воздух, необходимо, чтобы архимедова сила (выталкивающая) FА, действующая на шар, была больше силы тяжести Fтяж, т. е. FА > Fтяж.

По мере поднятия шара вверх архимедова сила, действующая на него, уменьшается (FА = gρV), так как плотность верхних слоев атмосферы меньше, чем у поверхности Земли. Чтобы подняться выше, с шара сбрасывается специальный балласт (груз) и этим облегчает шар. В конце концов шар достигает своей своей предельной высоты подъема. Для спуска шара из его оболочки при помощи специального клапана выпускается часть газа.

В горизонтальном направлении воздушный шар перемещается только под действием ветра, поэтому он называется аэростатом (от греч аэр — воздух, стато — стоящий). Для исследования верхних слоев атмосферы, стратосферы еще не так давно применялись огромные воздушные шары — стратостаты.

До того как научились строить большие самолеты для перевозки по воздуху пассажиров и грузов, применялись управляемые аэростаты — дирижабли. Они имеют удлиненную форму, под корпусом подвешивается гондола с двигателем, который приводит в движение пропеллер.

Воздушный шар не только сам поднимается вверх, но может поднять и некоторый груз: кабину, людей, приборы. Поэтому для того, чтобы узнать, какой груз может поднять воздушный шар, необходимо определить его подъемную силу.

Пусть, например, в воздух запущен шар объемом 40 м3, наполненный гелием. Масса гелия, заполняющая оболочку шара, будет равна:
mГе = ρГе·V = 0,1890 кг/м3 · 40 м3 = 7,2 кг,
а его вес равен:
PГе = g·mГе ; PГе = 9,8 Н/кг · 7,2 кг = 71 Н.
Выталкивающая же сила (архимедова), действующая на этот шар в воздухе, равна весу воздуха объемом 40 м3, т. е.
FА = g·ρвоздV; FА = 9,8 Н/кг · 1,3 кг/м3 · 40 м3 = 520 Н.

Значит, этот шар может поднять груз весом 520 Н — 71 Н = 449 Н. Это и есть его подъемная сила.

Шар такого же объема, но наполненный водородом, может поднять груз 479 Н. Значит, подъемная сила его больше, чем шара, наполненного гелием. Но все же чаще используют гелий, так как он не горит и поэтому безопаснее. Водород же горючий газ.

Гораздо проще осуществить подъем и спуск шара, наполненного горячим воздухом. Для этого под отверстием, находящимся в нижней части шара, располагается горелка. При помощи газовой горелки можно регулировать температуру воздуха внутри шара, а значит, его плотность и выталкивающую силу. Чтобы шар поднялся выше, достаточно сильнее нагреть воздух в нем, увеличив пламя горелки. При уменьшении пламени горелки температура воздуха в шаре уменьшается, и шар опускается вниз.

Можно подобрать такую температуру шара, при которой вес шара и кабины будет равен выталкивающей силе. Тогда шар повиснет в воздухе, и с него будет легко проводить наблюдения.

По мере развития науки происходили и существенные изменения в воздухоплавательной технике. Появилась возможность использования новых оболочек для аэростатов, которые стали прочными, морозоустойчивыми и легкими.

Достижения в области радиотехники, электроники, автоматики позволили сконструировать беспилотные аэростаты. Эти аэростаты используются для изучения воздушных течений, для географических и медико-биологических исследований в нижних слоях атмосферы.

Ссылки

  • Уроки по физике за 7 класс по школьной программе

Чему Равен Вес Тела В Жидкости
Вес тела,полностью погружённого в жидкость,в 2 раза меньше веса этого тела в воздухе.Сравните плотность тела и плотность жидкости. Вес тела в жидкости равен разнице двух прямо противоположных сил: силы тяжести и силы Архимеда.Fт = m*g, где m — масса тела (m = ρт*V, ρт — плотность тела, V — объем тела), g — ускорение свободного падения.Fa = ρж*g*V, где ρж — плотность жидкости.Вес тела в воздухе:Р1 = Fт = ρт*V*g.Вес тела в жидкости:Р2 = Fт — Fa = ρт*V*g — ρж*g*V.Так как Р1/Р2 = 2, тоρт*V*g/(ρт*V*g — ρж*g*V) = 2;ρт/(ρт — ρж) = 2;ρт = 2ρт — 2ρж;ρт = 2ρж.

Копировать с других сайтов запрещено. Стикеры и подарки за такие ответы не начисляются. Используй свои знания. :)Публикуются только развернутые объяснения. Ответ не может быть меньше 50 символов!

Читать подробнее: Вес тела,полностью погружённого в жидкость,в 2 раза меньше веса этого тела в воздухе.Сравните плотность тела и плотность

Чему равна масса тела в воде?

В воде вес тела Рв определяется формулой: Рв = Р — Fарх. Р = Рв + Fарх. m = (Рв + Fарх)/g. m = (2 Н + 0,5 Н)/9,8 м/с 2 = 0,255 кг.

Чему равен вес тела?

Чему равен вес тела, массой 10 кг?

Для нахождения значения веса указанного тела, необходимо использовать формулу: P = mт * g.Постоянные и переменные: mт — масса указанного тела (mт = 10 кг); g — ускорение свободного падения (g = 9,81 м/с 2 ).Вычисление: P = mт * g = 10 * 9,81 = 98,1 Н.Ответ: Вес указанного тела составляет 98,1 Н.

Знаешь ответ? Как написать хороший ответ? Будьте внимательны!

Копировать с других сайтов запрещено. Стикеры и подарки за такие ответы не начисляются. Используй свои знания. :)Публикуются только развернутые объяснения. Ответ не может быть меньше 50 символов!

Читать подробнее: Чему равен вес тела, массой 10 кг?

Сколько человек весит под водой?

Жидкость — Это 40-70% веса нашего тела. Причем, в мужчинах жидкости «содержится» больше, чем в женщинах. Если вы весите 68 кг и вы мужчина — то жидкости в вас может быть от 34 до 48 кг. Если вы женщина с тем же весом — то чуть меньше — от 27 до 41 кг воды.

Исключите углеводы — вода будет уходить, поскольку начнет расходоваться гликоген, запасенный в мышцах. Будете есть много соленого, вода будет задерживаться. Жидкость теряется ночью (до 1,4 кг за ночь), поэтому рекомендуется взвешиваться именно по утрам. В это время вы весите меньше всего.

Как найти вес тела формула?

По какой формуле можно определить вес тела? Формула, по которой определяется вес тела, записывается следующим образом: Р=mg, где Р-вес тела, m-масса тела, g-ускорение свободного падения. При решении задач, когда не требуется большой точности, g~9,8 Н/кг округляют до 10 Н/кг. Знаешь ответ? Как написать хороший ответ? Будьте внимательны!

Копировать с других сайтов запрещено. Стикеры и подарки за такие ответы не начисляются. Используй свои знания. :)Публикуются только развернутые объяснения. Ответ не может быть меньше 50 символов!

Читать подробнее: По какой формуле можно определить вес тела?

Как посчитать вес тела в жидкости?

Вес тела,полностью погружённого в жидкость,в 2 раза меньше веса этого тела в воздухе.Сравните плотность тела и плотность жидкости. Вес тела в жидкости равен разнице двух прямо противоположных сил: силы тяжести и силы Архимеда.Fт = m*g, где m — масса тела (m = ρт*V, ρт — плотность тела, V — объем тела), g — ускорение свободного падения.Fa = ρж*g*V, где ρж — плотность жидкости.Вес тела в воздухе:Р1 = Fт = ρт*V*g.Вес тела в жидкости:Р2 = Fт — Fa = ρт*V*g — ρж*g*V.Так как Р1/Р2 = 2, тоρт*V*g/(ρт*V*g — ρж*g*V) = 2;ρт/(ρт — ρж) = 2;ρт = 2ρт — 2ρж;ρт = 2ρж.

Копировать с других сайтов запрещено. Стикеры и подарки за такие ответы не начисляются. Используй свои знания. :)Публикуются только развернутые объяснения. Ответ не может быть меньше 50 символов!

Читать подробнее: Вес тела,полностью погружённого в жидкость,в 2 раза меньше веса этого тела в воздухе.Сравните плотность тела и плотность

Как найти массу тела в жидкости?

Вы замечали, что предметы в воде становятся легче? Существует много сказок и легенд, когда смекалистые герои поднимали вещи, которые на суше поднять было невозможно. Мы знаем, что сила тяжести не изменяется, но вес тела может зависеть от множества причин. Когда тело погружается в воду, его сила тяжести не изменяется, но появляется новая сила (открытая Архимедом), которая уменьшает вес этого тела.

Вес тела в воздухе: P = mg, Вес тела направлен вниз.Архимедова сила: F А = m ж ⋅ g, Сила направлена вверх.Вес тела в жидкости: P 1 = P − F А = mg − m ж g, Результирующая сил (1) и (2).

Тело, погружённое в жидкость, уменьшается в весе пропорционально весу вытесненной жидкости. Пример: определить, сколько весит в воде стеклянная пластина объёмом (1,5) дм³.

Дано Решение
V = 1,5 д м 3 = 0,0015 м 3 ρ ж = 1000 кг / м 3 ρ т = 2600 кг / м 3 g = 9, 8 Н / кг _ P 1 = m т g − m ж g m т = ρ т ⋅ V ; m ж = ρ ж ⋅ V ⇒ ⇒ P 1 = ρ т Vg − ρ ж Vg = Vg ρ т − ρ ж P 1 = 0,0015 м 3 ⋅ 9, 8 Н / кг ⋅ 2600 кг / м 3 − 1000 кг / м 3 ≈ 24 Н

Ответ: стеклянная пластина в воде весит (24) Н.

Как изменяется вес в воде?

Вес тела в воде и в воздухе Вес тела в воде и в воздухе На весах закреплены одинаковые грузы. Один из них погружен в воду. Почему тот груз, который весит в воздухе перевешивает? Причина заключается в том, что на грузы действует выталкивающая (архимедова) сила.

Эта сила направлена против силы тяжести и действует тогда, когда тело окружено какой-нибудь средой (т.е. в вакууме она действовать не будет). Чем больше плотность среды и чем больше объём погруженного туда тела, тем больше и архимедова сила. Вода намного плотнее воздуха, поэтому в воде архимедова сила больше, чем в воздухе.

Соответственно, вес груза в воде меньше. Широко известна легенда о том, как Архимед сделал свое открытие выталкивающей силы, принимая ванну. Но мало кто знает, почему это открытие было так важно для Архимеда. Оказывается, он догадался, как измерить объем короны царя Гиерона, чтобы определить, из чего она сделана.

Как измеряется масса тела?

Единицы измерения массы — Килограмм является одной из семи основных единиц СИ, По современному определению, его величина выражается через величины трёх выбранных физических постоянных: постоянная Планка, скорость света и частота определённого электронного перехода. В Международной системе единиц (СИ) масса измеряется в килограммах,

Единицей измерения массы в системе СГС является грамм ( 1 ⁄ 1000 килограмма). Вообще говоря, в любой системе измерения выбор основных (первичных) физических величин, их единиц измерения и их числа произволен — зависит от принимаемого соглашения и масса не всегда входит в их состав — так в системе МКГСС единица массы была производной единицей и измерялась в кГс ·с²/м (называлась «техническая единица массы» или « инерта »).

В атомной физике и химии принято сравнивать массу с относительной атомной массой ( а.е.м.), в физике твёрдого тела — с массой электрона ( Атомная система единиц ), в физике элементарных частиц массу измеряют в электронвольтах, Кроме этих единиц, используемых в науке, существует большое разнообразие исторических единиц измерения массы, которые сохранили свою отдельную сферу использования: фунт, унция, карат, тонна и др.

  • В астрофизике единицей для сравнения масс небесных тел служит масса Солнца,
  • В некоторых естественных системах единиц в качестве единицы массы используются массы элементарных частиц: электрона или протона,
  • В планковской системе единиц, также относящейся к естественным системам, единицей массы является планковская масса,

Массы очень мелких частиц могут быть определены с помощью величины, обратной к комптоновской длине волны : 1 см -1 ≈ 3,52⋅10 -41 кг, Масса очень большой звезды или чёрной дыры может быть отождествлена с её гравитационным радиусом : 1 см ≈ 6,73⋅10 24 кг,

Как записать вес тела?

Загрузить PDF Загрузить PDF Вес — сила, с которой тело действует на опору (или другой вид крепления), возникающая в поле силы тяжести. Масса связана с энергией и импульсом тела и эквивалентна энергии его покоя. Масса не зависит от силы тяжести (точнее от ускорения свободного падения).

  1. 1 Для вычисления веса используйте формулу Р = mg. Вес — это сила, с которой тело действует на опору, и его можно рассчитать, зная массу тела., где Р — вес тела (измеряется в ньютонах, Н), m, g — ускорение свободного падения (измеряется в м/с 2 ).
    • Так как вес является силой, эту формулу можно записать и как F = mg,
    • P или F — соответственно, вес или сила (измеряется в ньютонах, Н ).
    • m — масса тела (измеряется в килограммах, кг ).
    • g — ускорение свободного падения (измеряется в метрах на секунду в квадрате, м/с 2 ).
    • Ускорение свободного падения на поверхности Земли равно 9,8 м/с 2, Это стандартное значение, принятое в международной системе единиц.
  2. 2 Определите массу тела. Так как ускорение свободного падения — это стандартная величина, то необходимо знать массу тела, чтобы найти его вес. Масса должна быть выражена в килограммах.
  3. 3 Узнайте величину ускорения свободного падения. На Земле, как уже было сказано выше, g = 9,8 м/с 2, В других местах вселенной ускорение гравитации меняется. Если в задаче требуется найти вес тела на другой планете (или другом космическом объекте), выясните ускорение свободного падения на этом объекте.
    • Ускорение свободного падения на поверхности Луны примерно равно 1,622 м/с 2 (примерно в 6 раз меньше, чем на поверхности Земли). Поэтому ваш вес на Луне будет в 6 раз меньше вашего земного веса.
    • Ускорение свободного падения на Солнце примерно равно 274,0 м/с 2 (примерно в 28 раз больше, чем на Земле). Поэтому ваш вес на Солнце будет в 28 раз больше вашего земного веса (если конечно вы выживете на Солнце, что еще не факт!).
  4. 4 Подставьте значения в формулу. В формулу P=mg подставьте известные значения массы и ускорения свободного падения и найдите вес тела (измеряется в ньютонах, Н ). Реклама
  1. 1 Задача № 1. Найдите вес тела массой 100 кг на поверхности Земли.
    • В этой задаче m = 100 кг, g = 9,8 м/с 2 (так как нужно найти вес тела на Земле).
    • Р = 100 * 9,8 = 980 Н.
    • Вы решили задачу. Ответ: вес тела массой 100 кг на поверхности Земли равен 980 Н, или Р = 980 Н.
  2. 2 Задача № 2. Найдите вес тела массой 40 кг на поверхности Луны.
    • В этой задаче m = 40 кг, g = 1,6 м/с 2 (так как нужно найти вес тела на Луне).
    • Р = 40 * 1,6 = 64 Н.
    • Вы решили задачу. Ответ: вес тела массой 40 кг на поверхности Луны равен 64 Н, или Р = 64 Н.
  3. 3 Задача № 3. Найдите массу тела, которое на поверхности Земли весит 549 Н.
    • В этой задаче Р = 549 Н, g = 9,8м/с 2 (так как дан вес тела на Земле).
    • Выведите m из формулы P = mg. Получится m = P/g. Подставьте известные значения: m = 549/9,8 = 56 кг.
    • Вы решили задачу. Ответ: масса тела, которое весит 549 Н (на поверхности Земли) равна 56 кг, или m = 56 кг.

    Реклама

  1. 1 Не путайте массу и вес. Самая распространенная ошибка — перепутать вес и массу (что немудрено, ведь в повседневной жизни мы обычно называем массу весом). Но в физике все не так. Запомните, масса — это постоянное свойство объекта, то, сколько в нем вещества (килограммов), где бы он ни находился. Вес — это сила, с которой объект всеми своими килограммами давит на поверхность, и эта сила на разных небесных телах будет различной.
  2. 2 Используйте правильные единицы измерения. В задачах по физике вес или силу измеряют в ньютонах (Н), ускорение свободного падения — в метрах на секунду в квадрате (м/с 2 ), а массу — в килограммах (кг). Если для какой-либо из этих величин вы возьмете не ту единицу измерения, воспользоваться формулой будет нельзя, Если масса в условиях задачи указана в граммах или тоннах, не забудьте перевести ее в килограммы.
  3. 3 Проверяйте единицы измерения. Если вы решаете сложную задачу в несколько действий, следите, чтобы у вас получались правильные единицы. Запомните, что 1 ньютон эквивалентен 1 (кг*м)/с 2, При необходимости подставляйте этот эквивалент вместо ньютонов, чтобы путем сокращения получить нужную единицу.
    • Задача. Вес Игоря на Земле составляет 880 ньютонов. Какова его масса?
    • масса = (880 Н)/(9,8 м/с 2 )
    • масса = 90 Н/(м/с 2 )
    • масса = (90 кг*м/с 2 )/(м/с 2 )
    • Сокращаем м/с 2 и получаем: масса = 90 кг.
    • Масса должна быть выражена в килограммах, а значит, вы правильно решили задачу.

    Реклама

  • Ньютон — это единица измерения силы в международной системе единиц СИ. Нередко сила выражается в килограмм-силах, или кгс (в системе единиц МКГСС). Эта единица очень удобна для сравнения весов на Земле и в космосе.
  • 1 кгс = 9,8166 Н.
  • Разделите вес, выраженный в ньютонах, на 9,80665.
  • Вес космонавта, который «весит» 101 кг (то есть его масса равна 101 кг), составляет 101,3 кгс на Северном полюсе и 16,5 кгс на Луне.
  • Международная система единиц СИ — система единиц физических величин, которая является наиболее широко используемой системой единиц в мире.

Сколько весит 1 литр жира?

Сколько килограммов можно сбросить после операции? — За одну операцию откачивают, как правило, не больше 2,5 литра жира. Один литр весит около 900 граммов. То есть глобального похудения не будет.

Сколько жидкости на кг?

Сколько воды нужно пить в течение дня? — Здоровому взрослому человеку требуется около 35 мл воды в день на килограмм массы тела — в соответствии с общими рекомендациями научных организаций. Для человека весом 50 кг необходимо 1,7 л, 60 кг — 2,1 л, 70 кг — 2,4 л, 80 кг — 2,8 л воды каждый день.

  • Основное правило: чем больше вы весите, тем больше воды нужно пить.
  • Думаете, невозможно выпить слишком много воды? Слишком большое количество воды так же опасно для организма, как и обезвоживание.
  • Рекомендуемый суточный объем отражает количество воды, которое ваши почки и сердце могут выдержать.
  • Ежедневный объем воды также зависит от возраста, диеты, уровня активности и климата.

Активный ребенок, который гуляет на улице весь день, должен пить больше воды, чем ребенок, проводящий весь день в своей комнате за компьютером или книгами. Тем не менее, здоровому ребенку рекомендуется выпивать около 1,1 литра в день — независимо от чувства жажды.

Сколько воды в теле человека весом 60 кг?

Вычисли и запиши, сколько воды в теле человека, если его масса составляет. a)60кг-, б) 90кг-, Известно, что содержание воды в теле человека зависит от его возраста. Чем старше человек, тем меньше процентное содержание воды в его организме. Тело взрослого человека на 65% состоит из воды.

Копировать с других сайтов запрещено. Стикеры и подарки за такие ответы не начисляются. Используй свои знания. :)Публикуются только развернутые объяснения. Ответ не может быть меньше 50 символов!

Читать подробнее: Вычисли и запиши, сколько воды в теле человека, если его масса составляет. a)60кг-, б) 90кг-,

В чем измеряется вес тела?

У этого термина существуют и другие значения, см. Вес (значения), Не следует путать с массой, Вес — сила, с которой тело действует на опору (или подвес, или другой вид крепления), препятствующую падению, возникающая в поле сил тяжести, Единица измерения веса в Международной системе единиц (СИ) — ньютон, иногда используется единица СГС — дина,

Как найти вес через объем?

Вес можно рассчитать по формуле: m=V*p, где р – плотность, V – объем материала. Например, 10 м3 речного песка весят 13 тонн.

Чему равно g?

Текущая версия страницы пока не проверялась опытными участниками и может значительно отличаться от версии, проверенной 19 августа 2022 года; проверки требует 1 правка,

Ускорение свободного падения на поверхности некоторых небесных тел, м/с 2 и g

Земля 9,81 м/с 2 1,00 g Солнце 273,1 м/с 2 27,85 g
Луна 1,62 м/с 2 0,165 g Меркурий 3,70 м/с 2 0,378 g
Венера 8,88 м/с 2 0,906 g Марс 3,86 м/с 2 0,394 g
Юпитер 24,79 м/с 2 2,528 g Сатурн 10,44 м/с 2 1,065 g
Уран 8,86 м/с 2 0,903 g Нептун 11,09 м/с 2 1,131 g
Эрида 0,82 ± 0,02 м/с 2 0,084 ± 0,002 g Плутон 0,617 м/с 2 0,063 g

Ускоре́ние свобо́дного паде́ния ( ускорение силы тяжести ) — ускорение, придаваемое телу силой тяжести, при исключении из рассмотрения других сил. В соответствии с уравнением движения тел в неинерциальных системах отсчёта ускорение свободного падения численно равно силе тяжести, воздействующей на объект единичной массы,

Ускорение свободного падения на поверхности Земли g (обычно произносится как «же» ) варьируется от 9,780 м/с² на экваторе до 9,82 м/с² на полюсах, Стандартное («нормальное») значение, принятое при построении систем единиц, составляет 9,80665 м/с², Стандартное значение g было определено как «среднее» в каком-то смысле на всей Земле: оно примерно равно ускорению свободного падения на широте 45,5° на уровне моря,

В приблизительных расчётах его обычно принимают равным 9,81, 9,8 или более грубо 10 м/с².

Сколько воды в теле человека массой 60 кг?

Сколько воды в теле человека если его масса составляет 60 кг 42 кг 100:70=42(кг) ответ:42 килограмма в теле человека. (хорошего дня!!!) Содержание воды в теле человека зависит от многих факторов, в том числе и от возраста. Так, ребенок почти на 90% состоит из воды. Человек среднего возраста — уже на 70%.Предположим, нам надо рассчитать количество воды в теле человека среднего возраста, если его вес составлет 60 кг.Расчет будет выглядеть следующим образом:60:100х70=42 кг.Ответ: в теле человека среднего возраста весом 60 кг содержится 42 кг воды.

Копировать с других сайтов запрещено. Стикеры и подарки за такие ответы не начисляются. Используй свои знания. :)Публикуются только развернутые объяснения. Ответ не может быть меньше 50 символов!

Читать подробнее: Сколько воды в теле человека если его масса составляет 60 кг

Когда тело массой 10 кг опустили в воду оно потеряло в весе 25 Н?

Вес тела в воздухе найдем по формуле: P = m * g = 10 * 10 = 100 Ньютон. По условию задачи, при погружении в воду тело потеряло в весе 25 Ньютон, то есть: P1 = P — 25 = 100 — 25 = 75 Ньютон.

Сколько воды в теле если его масса составляет 60 кг?

Вычисли и запиши, сколько воды в теле человека, если его масса составляет. a)60кг-, б) 90кг-, Известно, что содержание воды в теле человека зависит от его возраста. Чем старше человек, тем меньше процентное содержание воды в его организме. Тело взрослого человека на 65% состоит из воды.

Копировать с других сайтов запрещено. Стикеры и подарки за такие ответы не начисляются. Используй свои знания. :)Публикуются только развернутые объяснения. Ответ не может быть меньше 50 символов!

Читать подробнее: Вычисли и запиши, сколько воды в теле человека, если его масса составляет. a)60кг-, б) 90кг-,

Чему равна масса и объем тела?

Содержание:

  • § 1 Расчет массы и объема вещества по его плотности
  • § 2 Решение задач
  • § 3 Важно запомнить

§ 1 Расчет массы и объема вещества по его плотности В этом уроке мы изучим, как можно определить массу и объем тела, если известна плотность вещества. Плотность – скалярная физическая величина, показывающая, чему равна масса вещества, взятого в объеме 1 м3, и равная отношению массы тела к его объему: p = m : v.

Из формулы плотности следует, что масса тела равна произведению плотности вещества на объем этого тела: m = ρ · V. Чтобы вычислить объем тела, нужно массу тела разделить на его плотность: v = m : p. Для правильного решения задач нужно уметь верно переводить единицы измерения величин в Международную систему единиц: 1 г = 0,001 кг, 1 л = 1 дм3 = 0,001 м3, 1 см3 = 0,000 001 м3, 1 г/см3 = 1000 кг/м3.

§ 2 Решение задач Какова масса подсолнечного масла в бутылке объемом 3 л, если плотность масла равна 930 кг/м3? Запишем условие задачи. Нам известны объем бутылки (обозначается буквой V) 3 л, и плотность подсолнечного масла (обозначается буквой ρ) 930 кг/м3. Сколько штук строительного кирпича размером 250 мм х 120 мм х 65 мм допускается перевозить на автомашине грузоподъемностью 4 т? Плотность кирпича 1800 кг/м3. Запишем условие задачи и выразим данные в Международной системе единиц. Известны размеры кирпича: длина а = 250 мм = 0,25 м, ширина b= 120 мм = 0,12 м, высота с = 60 мм = 0,06 м, плотность кирпича ρ = 1800 кг/м3, грузоподъемность – наибольшая масса груза, которую может перевезти автомобиль – m = 4 т = 4000 кг.

Найти количество кирпичей – обозначим латинской буквой N. Решение: Количество кирпичей можно найти, поделив общую массу всех кирпичей на массу одного кирпича: N = m/m1. Чтобы найти массу одного кирпича, нужно плотность умножить на его объем: m1 = ρ · V. Кирпич имеет форму прямоугольного параллелепипеда, следовательно, его объем равен произведению длины, ширины и высоты кирпича.

Подставим числовые значения известных величин и вычислим. Объем кирпича равен 0,0018 м3. Масса одного кирпича m1 равна 1800 кг/м3, умножим на 0,0018 м3, равно 3,24 кг. Тогда число кирпичей равно N 4000 кг, разделим на 3,24 кг и получим 1234, 567 штук или число целых кирпичей 1234 штуки. Медный шар имеет массу 840 г при объеме 120 см3. Сплошной этот шар или полый? Плотность меди 8900 кг/м3. Запишем условие задачи. Известна масса шара m 840 г, что в системе СИ составляет 0,84 кг, объем шара V=120 см3, в СИ 0,00 012 м3, плотность меди ρ = 8900 кг/м3.

  1. Определить, сплошной шар или содержит внутри пустое пространство? Решение.
  2. Представим, что на рычажных весах лежат два медных шара, один сплошной, второй содержит внутри пустое пространство, то есть полый шар.
  3. Если у них массы одинаковы, то объем полого шара должен быть больше, чем объем сплошного шара (рис 1).

Определим, каков объем шара, состоящего полностью из меди. Если объем окажется равным 120 см3, то шар сплошной и пустот не содержит. Если же вычисленный объем окажется меньше 120 см3, значит, внутри есть полость. Чтобы найти объем сплошного медного шара, массу шара разделим на его плотность. § 3 Важно запомнить Плотность – скалярная физическая величина, показывающая, чему равна масса вещества, взятого в объеме 1 м3, и равная отношению массы тела к его объему: p = m : v. Масса тела равна произведению плотности вещества на объем этого тела: m = ρ · V. Чтобы вычислить объем тела, нужно массу тела разделить на его плотность: V = m : p. Список использованной литературы:

  1. Волков В.А. Поурочные разработки по физике: 7 класс. – 3-е изд. – М.: ВАКО, 2009. – 368 с.
  2. Волков В.А. Тесты по физике: 7-9 классы. – М.: ВАКО, 2009. – 224 с. – (Мастерская учителя физики).
  3. Кирик Л.А. Физика -7. Разноуровневые самостоятельные и контрольные работы. — М.: Илекса, 2008. – 192 с.
  4. Контрольно-измерительные материалы. Физика: 7 класс / Сост. Зорин Н.И. – М.: ВАКО, 2012. – 80 с.
  5. Марон А.Е., Марон Е.А. Физика.7 Дидактические материалы. – М.: Дрофа, 2010. – 128 с.
  6. Перышкин А.В. Физика.7 класс — М.: Дрофа, 2011.
  7. Тихомирова С.А. Физика в пословицах и поговорках, стихах и прозе, сказках и анекдотах. Пособие для учителя. – М.: Новая школа, 2002. – 144 с.

Использованные изображения:

Содержание:

Гидростатическое взвешивание:

На этом принципе основан метод так называемого гидростатического взвешивания. Если в мензурку опустить деревянный брусок, то он будет плавать, но уровень воды поднимется. Объем этой воды равен объему погруженной части бруска, а ее вес — весу бруска. Зная объем и плотность воды, можно рассчитать вес воды и вес тела. Для случая, когда тело тяжелее воды, изготавливают специальный поплавок, дающий возможность телу плавать по поверхности воды.

Гидростатическое взвешивание

Гидростатическое взвешивание — это метод измерения плотности жидкости или твёрдого тела, основанный на законе Архимеда. Плотность твёрдых тел определяют методом двойного взвешивания тела: сначала в воздухе, а потом в жидкости, плотность которой известна. Если определяют плотность жидкости, то в ней взвешивают тело известной массы и объёма.

Если исследуемое сплошное твёрдое тело тонет в воде, то для выполнения задания нужен лишь лабораторный динамометр (или равноплечие весы) и сосуд с водой.

Сначала определяют вес Р исследуемого тела в воздухе: Гидростатическое взвешивание в физике - формулы и определения с примерами

Потом твёрдое тело погружают в сосуд с жидкостью, плотность которой Гидростатическое взвешивание в физике - формулы и определения с примерами известна (в случае использования дистиллированной или чистой воды Гидростатическое взвешивание в физике - формулы и определения с примерами = 1000 Гидростатическое взвешивание в физике - формулы и определения с примерами), и определяют вес тела Гидростатическое взвешивание в физике - формулы и определения с примерами в жидкости, который по закону Архимеда меньше веса тела в воздухе на значение силы Архимеда Гидростатическое взвешивание в физике - формулы и определения с примерами     Гидростатическое взвешивание в физике - формулы и определения с примерами отсюда Гидростатическое взвешивание в физике - формулы и определения с примерами, или Гидростатическое взвешивание в физике - формулы и определения с примерами.

Из этой формулы можно определить плотность жидкости, если она неизвестна, а объём тела известен:Гидростатическое взвешивание в физике - формулы и определения с примерами

Объём жидкости, вытесненной телом, равен объёму тела, но

поскольку Гидростатическое взвешивание в физике - формулы и определения с примерами то Гидростатическое взвешивание в физике - формулы и определения с примерами . Подставим это в выражение

для архимедовой силы, получим  Гидростатическое взвешивание в физике - формулы и определения с примерами , отсюда и вытекает искомая формула для определения плотности вещества твёрдого тела: Гидростатическое взвешивание в физике - формулы и определения с примерами.

Пример №1

Купаясь в реке с илистым дном, можно заметить, что ноги больше вязнут на мелких местах, чем на глубоких. Объясните, почему.

Ответ: так как на глубоких местах действует большая выталкивающая сила.

Пример №2

Определите, какая архимедова сила действует на тело объёмом

5 м3 , погружённое полностью в воду?

Дано: 

V = 5м3

Гидростатическое взвешивание в физике - формулы и определения с примерами= 9,81 Гидростатическое взвешивание в физике - формулы и определения с примерами

Гидростатическое взвешивание в физике - формулы и определения с примерами= 1000 Гидростатическое взвешивание в физике - формулы и определения с примерами

FА = 49,05 кН

FА = ?

Решение:

По формуле Гидростатическое взвешивание в физике - формулы и определения с примерами определим архимедову силу:

Гидростатическое взвешивание в физике - формулы и определения с примерами

Ответ: = 49,05 кН.

Пример №3

Нужно ли учитывать загрузку судна при переходе его из моря в реку? Догружать или разгружать нужно судно, чтобы его осадка была не глубже ватерлинии?

Ответ: при переходе судна из моря в реку нужно учитывать загрузку судна, так как плотность воды уменьшается. Судно нужно разгружать.

  • Заказать решение задач по физике

Теоретические сведения

Гидростатическое взвешивание издавна применяется для определения плотности различных веществ. Для этого используют закон Архимеда. Плотность твердых тел определяют двойным взвешиванием: сначала тело взвешивают в воздухе (при этом в большинстве случаев выталкивающей силой воздуха пренебрегают), а потом — в жидкости, плотность которой известна (например, в воде). Рассмотрим методы определения плотности.

1. Если исследуемое тело тонет в воде (его плотность рт превышает плотность воды рв), то в таком случае используют динамометр и стакан с водой.

Сначала исследуемое тело взвешивают в воздухе (рис. 120, а): Гидростатическое взвешивание в физике - формулы и определения с примерами

В этом случае архимедовой силой, действующей на тело в воздухе, можно пренебречь, так как плотность воздуха намного меньше плотности тела и воды.

Гидростатическое взвешивание в физике - формулы и определения с примерами

Потом тело опускают в стакан с водой (рис. 120, б), плотность воды известна Гидростатическое взвешивание в физике - формулы и определения с примерами В этом случае на тело, кроме сил тяжести Гидростатическое взвешивание в физике - формулы и определения с примерами и упругости пружины динамометра Гидростатическое взвешивание в физике - формулы и определения с примерами, действует сила Архимеда Гидростатическое взвешивание в физике - формулы и определения с примерами:

Гидростатическое взвешивание в физике - формулы и определения с примерами

Таким образом, 

Гидростатическое взвешивание в физике - формулы и определения с примерами

2. Для измерения плотности неизвестной жидкости можно воспользоваться также телом, которое не тонет в воде и исследуемой жидкости, например карандашом или другим телом правильной формы. Чтобы карандаш в жидкости занимал вертикальное положение, к его нижнему концу можно приколоть несколько кнопок или намотать несколько витков проволоки.

Гидростатическое взвешивание в физике - формулы и определения с примерами

Если карандаш плавает в воде (рис. 121, а), то сила тяжести Гидростатическое взвешивание в физике - формулы и определения с примерами действующая на него, равна силе Архимеда Гидростатическое взвешивание в физике - формулы и определения с примерами. В этом случае

Гидростатическое взвешивание в физике - формулы и определения с примерами

где Гидростатическое взвешивание в физике - формулы и определения с примерами — объем тела, a Гидростатическое взвешивание в физике - формулы и определения с примерамиГидростатическое взвешивание в физике - формулы и определения с примерами — объем вытесненной телом воды (объем погруженной части тела).

Если тело опустить в неизвестную жидкость (рис. 121, б), плотность которой Гидростатическое взвешивание в физике - формулы и определения с примерами, то

Гидростатическое взвешивание в физике - формулы и определения с примерами

С этого уравнения имеем

Гидростатическое взвешивание в физике - формулы и определения с примерами

Отсюда

Гидростатическое взвешивание в физике - формулы и определения с примерами

3.    Плотность неизвестной жидкости можно определить с помощью резиновой нити, тела, которое тонет в воде и неизвестной жидкости, и линейки. Последовательность действий при этом показана на рисунке 122.

Гидростатическое взвешивание в физике - формулы и определения с примерами

Длина резиновой нити (или пружины) без нагрузки Гидростатическое взвешивание в физике - формулы и определения с примерами (рис. 122, а). Если к ней прикрепить тело в воздухе (рис. 122, б), то сила тяжести Гидростатическое взвешивание в физике - формулы и определения с примерами будет равна по значению силе упругости Гидростатическое взвешивание в физике - формулы и определения с примерами возникшей в нити. Тело будет в состоянии равновесия.

Теперь, если тело опустить в воду (рис. 122, в), то на него будет действовать еще сила Архимеда:

Гидростатическое взвешивание в физике - формулы и определения с примерами

Опустим тело в жидкость, плотность которой нужно определить (рис. 122, г).

4.    Для определения плотности твердого тела или неизвестной жидкости можно использовать рычаг. Для этого нужно иметь две гирьки, плотность одной из них массой Гидростатическое взвешивание в физике - формулы и определения с примерами необходимо определить, рычаг, линейку, стаканы с водой и неизвестной жидкостью. Последовательность действий показана на рисунке 123.

Гидростатическое взвешивание в физике - формулы и определения с примерами

Для определения плотности тела используем формулу

Гидростатическое взвешивание в физике - формулы и определения с примерами

Для определения плотности неизвестной жидкости можно использовать формулу

Гидростатическое взвешивание в физике - формулы и определения с примерами

  • Воздухоплавание в физике
  • Машины и механизмы в физике
  • Коэффициент полезного действия (КПД) механизмов
  • Тепловые явления в физике
  • Барометры в физике
  • Жидкостные насосы в физике
  • Выталкивающая сила в физике
  • Условия плавания тел в физике 

Понравилась статья? Поделить с друзьями:
  • Как составить план пересказа текста 2 класс литературное чтение
  • Порядковый номер 345 не найден в библиотеке dll comctl32 dll как исправить
  • Найди мне канал как мама
  • Как найти тангенс между двумя прямыми
  • System indexoutofrangeexception как исправить